Научная статья на тему 'Наноэлектроника «Снизу - вверх»: эффекты Холла, измерение электрохимических потенциалов и транспорт спинов в модели НРФГ'

Наноэлектроника «Снизу - вверх»: эффекты Холла, измерение электрохимических потенциалов и транспорт спинов в модели НРФГ Текст научной статьи по специальности «Физика»

CC BY
84
18
i Надоели баннеры? Вы всегда можете отключить рекламу.
Журнал
ScienceRise
Область наук
Ключевые слова
НАНОЭЛЕКТРОНИКА / ЭФФЕКТЫ ХОЛЛА / ИЗМЕРЕНИЕ ХИМПОТЕНЦИАЛА / ТРАНСПОРТ СПИНОВ / МЕТОД НРФГ / ГРАФЕН / NANOELECTRONICS / HALL EFFECTS / CHEMICAL POTENTIAL MEASUREMENT / SPIN TRANSPORT / NEGF METHOD / GRAPHENE

Аннотация научной статьи по физике, автор научной работы — Кругляк Ю.А.

В концепции «снизу вверх» наноэлектроники рассматриваются эффекты Холла, методы измерения электрохимических потенциалов, подходы Ландауэра и Бюттекера, учет магнитного поля в методе неравновесных функций Грина (НРФГ), спиновый транспорт в формализме НРФГ в спинорном представлении, обсуждаются вращение магнитных контактов и спинов, роль спиновых гамильтонианов Зеемана и Рашбы, квантовый спиновый эффект Холла, вычисление спинового потенциала, четырех компонентный формат описания спинового транспорта

i Надоели баннеры? Вы всегда можете отключить рекламу.
iНе можете найти то, что вам нужно? Попробуйте сервис подбора литературы.
i Надоели баннеры? Вы всегда можете отключить рекламу.

The Hall effects, measurement of electrochemical potentials, the Landauer and Buttiker approaches, an account of magnetic field in the non-equilibrium Green’s functions (NEGF) method, spin transport by the NEGF method in the spinor representation, in particular, rotating magnetic contacts and spins, Zeeman and Rashba spin Hamiltonians, calculation of the spin potential, and four-component description of spin transport are discussed in the«bottom up» approach of modern nanoelectronics

Текст научной работы на тему «Наноэлектроника «Снизу - вверх»: эффекты Холла, измерение электрохимических потенциалов и транспорт спинов в модели НРФГ»

Ф1ЗИКО-МАТЕМАТИЧН1 НАУКИ

УДК 537.32

DOI: 10.15587/2313-8416.2015.51353

НАНОЭЛЕКТРОНИКА «СНИЗУ - ВВЕРХУ»: ЭФФЕКТЫ ХОЛЛА, ИЗМЕРЕНИЕ ЭЛЕКТРОХИМИЧЕСКИХ ПОТЕНЦИАЛОВ И ТРАНСПОРТ СПИНОВ В МОДЕЛИ НРФГ

©Ю. А. Кругляк

В концепции «снизу - вверх» наноэлектроники рассматриваются эффекты Холла, методы измерения электрохимических потенциалов, подходы Ландауэра и Бюттекера, учет магнитного поля в методе неравновесных функций Грина (НРФГ), спиновый транспорт в формализме НРФГ в спинорном представлении, обсуждаются вращение магнитных контактов и спинов, роль спиновых гамильтонианов Зе-емана и Рашбы, квантовый спиновый эффект Холла, вычисление спинового потенциала, четырехкомпо-нентный формат описания спинового транспорта

Ключевые слова: наноэлектроника, эффекты Холла, измерение химпотенциала, транспорт спинов, метод НРФГ, графен

The Hall effects, measurement of electrochemical potentials, the Landauer and Buttiker approaches, an account of magnetic field in the non-equilibrium Green's functions (NEGF) method, spin transport by the NEGF method in the spinor representation, in particular, rotating magnetic contacts and spins, Zeeman and Rashba spin Ham-iltonians, calculation of the spin potential, and four-component description of spin transport are discussed in the «bottom - up» approach of modern nanoelectronics

Keywords: nanoelectronics, Hall effects, chemical potential measurement, .spin transport, NEGF method, graphene

1. Введение и обзор литературы

Эдвин Холл в 1879 году обнаружил возникновение поперечной разности потенциалов при помещении тонкой золотой пластинки с постоянным током в магнитное поле [1]. В простейшем варианте эффект Холла проявляется следующим образом. Поместим 20 проводник с электрическим током в слабое магнитное поле В , направленное перпендикулярно к поверхности проводника - вдоль оси у на рис. 1.

Магнитное поле под действием силы Лоренца

Рис. 1. Магнитное поле, направленное перпендикулярно к проводнику с током, отклоняет электроны к краям проводника

е Ф - 5 F = — = -сп ■ х В

dt

(1)

отклоняет электроны, движущиеся со скоростью V от истока к стоку Б, от их траектории вдоль или против вектора скорости к боковым краям проводника (рис. 1). Критерием малости магнитного поля обычно служит условие, чтобы электроны не начали двигаться по циклоиде.

Поскольку больше электронов движется от истока к стоку, создается холловская разность потенциалов УИ в поперечном направлении х (рис. 2).

Рис. 2. Холловская разность потенциалов создается в поперечном направлении

Литературный обзор. Эффект Холла всегда вызывал значительный интерес. Он особенно возрос в 1980 году, когда К. фон Клитцинг обнаружил, что холловское сопротивление

Rh = Vl

i

(2)

принимает дискретные значения [2, 3], которые могут быть вычислены с помощью следующей простой формулы

Rh =

25812.80

k '

k = 1,2,3,...

(3)

Обнаруженное явление получило название целочисленного квантового эффекта Холла. Точность измерения сопротивления столь велика, что Национальное бюро стандартов США выбрало именно эффект Холла для калибровки сопротивлений [4].

К. фон Клитцинг показал [2, 3], что холловское сопротивление фактически измеряет отношение двух фундаментальных физических констант - постоянной Планка и квадрата заряда электрона

Rh = h■ р k = 1,2,3,..., q k

(4)

а через скорость света с связано с постоянной тонкой структуры

Ч

1

he 137.0359895

(5)

что имеет исключительно важное значение для метрологии фундаментальных констант физики.

Уже в 1982 году был экспериментально обнаружен дробный квантовый эффект Холла [5, 6]. В ферромагнетиках в холловском сопротивлении обнаружен вклад, зависящий непосредственно от намагниченности материала, причем этот вклад может быть намного больше нормального холловского сопротивления. Это явление получило название аномального эффекта Холла [7]. Известны и другие, не менее интересные проявления эффекта Холла - спиновый эффект Холла Дьяконова- Переля [8, 9], квантовый спиновый эффект Холла в графене [10], как и иные проявления действия магнитного поля на электрические цепи, как, например, невзаимные электрические цепи (non-reciprocity circuits), которые трудно интерпретировать в терминах обычных эффективных сопротивлений [11], что тем более интересно, поскольку появился новый класс материалов - топологические диэлектрики (topological insulators), которые как будто бы обладают невзаимностью даже в отсутствии магнитных полей [12]. Эти и другие смежные вопросы мы рассмотрим с разной степенью полноты с позиций концепции «снизу - вверх» современной наноэлектроники [13].

Рассмотрим подробнее обычный классический эффект Холла при низких магнитных полях. Одной из причин, вызвавших повышенный интерес к эффекту Холла, был тот факт, что холловское сопротивление имеет разный знак для n- и ^-проводников,

что открывает возможность экспериментально определять различие между ними. Этой же цели можно достичь, измеряя направление тока по проводнику между контактами, находящимися при разных температурах [14]. И в том и в другом случае обычно пользуются понятием «дырок» как носителей положительного заряда в ^-проводниках. Подобную трактовку нельзя признать удовлетворительной, поскольку и в п- и в ^-проводниках носителями тока являются только электроны.

В случае термоэлектрических измерений направление тока определяется наклоном плотности состояний Б(Е), то ли эта функция является растущей с ростом энергии (п-проводник), то ли падающей (р-проводник) [14]. Знак же холловского эффекта определяется знаком эффективной массы, определяемой по (75) из работы [15] как отношение импульса р к скорости ЛЕ/Лр . В результате, хотя сила Лоренца (1) одна и та же и для п- и для р-проводников, давая одно и то же значение ф/Л, результирующая МЛ имеет разные знаки для п- и р-проводников, закручивая электроны в последних в противоположном направлении, что и приводит к изменению знака хол-ловской разности потенциалов.

Обратимся к элементарной теории эффекта Холла, как она обычно излагается в учебниках по физике твердого тела, например [16]. Ток дается выражением

I = д (М / Ь)уа , (6)

где Ы/Ь есть линейная плотность электронов, а их дрейфовая скорость равна произведению подвижности электронов на электрическое поле вдоль длины проводника

V, =-Ц{У / Ь) . (7)

В состоянии равновесия поперечное поле компенсирует силу Лоренца

VH / W = vdB ,

(8)

так что используя (6) и (7) для холловского сопротивления имеем

R = V^ = B

H I q(N/LW) '

(9)

Это уравнение широко используется для определения поверхностной плотности электронов N/ЬW по наклону зависимости холловского сопротивления от магнитного поля.

Эта элементарная теория эффекта Холла излагается с теми или иными незначительными нюансами во всех учебниках. И везде фигурирует электрическое поле как в продольном, так и в поперечном направлении в качестве причины возникновения электрического тока. Согласиться с этим никак нельзя [15]. Согласно (9), холловское сопротивление зависит от электронной плотности по всему спектру энергий, что, конечно, не так. Как и любые другие транспортные коэффициенты [14], холловское сопротивление - это свойство фермиевской поверхности и

зависит оно только от электронов, энергия которых лежит в интервале приблизительно нескольких кТ вокруг значения Е = и [15].

Мы покажем, что холловское сопротивление для одномодового упругого резистора дается выражением

^ (Е) ^ 2ВШ

qD ( E )v( E ) p ( E )

(10)

которое нужно еще усреднить по энергии в интервале нескольких кТ вокруг значения Е = /и0, используя стандартную функцию теплового уширения ¥Т [15], а именно:

■t-ТdE i-

JL

dE

1

RH (E )"

(11)

Обратим внимание на то, что в общем случае нужно усреднять проводимость 1/Ян(Е), а не сопротивление ЯНЕ), поскольку все модовые каналы проводимости «работают» параллельно и находятся под одной и той же разностью потенциалов.

Уравнения (10), (11) сводятся к стандартному выражению (9), используя фундаментальное соотношение [15]

D ( E)v( E) p (E) = N (E) • d

(12)

для 2D проводника ^ = 2) и усредняя Щ(Е), как это было показано в [15]. Если же дисперсионное соотношение для изучаемого проводника не известно, то остается только пользоваться уравнениями (10) - (11).

В любом случае уравнение (11) свидетельствует о том, что эффект Холла не затрагивает все электроны при всех возможных энергиях. Есть, правда, одно явление, которое затрагивает все электроны при любых энергиях, - это существование равновесных токов в проводнике, помещенном в магнитное поле (рис. 3).

Равновесные

Рис. 3. Равновесные токи в проводнике, находящемся в магнитном поле

Однако же, неисчезающие токи, которые существуют даже в атоме водорода, помещенном в магнитное поле, не имеют никакого отношения к транспортным коэффициентам. Транспортные модели надлежит строить таким образом, чтобы неферми-евские токи элиминировались с самого начала. Подобная проблема по отношению к спиновым токам возникает даже без учета внешнего магнитного поля [17].

Далее мы рассмотрим расчет неравновесных электрохимических потенциалов внутри проводника, а также моделирование проводника в четырехтерми-нальном окружении с использованием уравнения Бюттикера [18, 19]. Однако, сначала конспективно рассмотрим общую динамику электронов в магнитном поле.

2. Почему отличаются между собой п- и р-проводники?

Почему холловское сопротивление имеет противоположные знаки для п- и ^-проводников? Основная причина состоит в том, что в п-проводниках скорость и импульс сонаправлены, а в ^-проводниках -антинаправлены; и поскольку V = dEldp, а в ^-проводниках энергия уменьшается с ростом импульса (рис. 4) [15].

Рис. 4. Графическая демонстрация разных знаков холловского сопротивления для п-проводников (слева) и ^-проводников (справа)

Чтобы увидеть какую роль играет различие в знаках скорости и импульса, обратимся к силе Лоренца (1). Для любого изотропного дисперсионного соотношения скорость и импульс коллинеарны (параллельны или антипараллельны) и пусть находятся, например, под углом в к продольной оси г проводника (рис. 5).

Рис. 5. Вектор скорости (импульса) направлен под углом в к оси проводника г (рис. 1).

Тогда

p = p cos0z + рътбх, v= rcos0z+rsinöx.

а после подстановки в (1) получим

дуБ Л р

(13)

(14)

Другими словами, угол 9 меняется линейно со временем, а векторы скорости и импульса вращаются с постоянной угловой скоростью ас . Однако, для пи ^-проводников вращение происходит в противоположных направлениях, поскольку отношение р / V у них имеет разные знаки. Это отношение определяется как масса (ур-е (75) в [15]) и является константой для параболической дисперсии (ур-е (78) в [15]), так что циклотронная частота

qvB

E=Uo

qB

(15)

Для линейной дисперсии (ур-е (79) в [15]) масса растет с энергией, так что циклотронная частота (15) уменьшается с ростом плотности носителей тока, что и наблюдается в графене [20].

Магнитное поле вращает электроны по круговой траектории с частотой сос . Если юс-т << 1, где

т - среднее время свободного вращения, то вращение никогда не заканчивается, что соответствует режиму наблюдения холловского сопротивления (9)-(11) в слабом магнитном поле. В режиме сильного магнитного поля с ' т>> 1 и наблюдается квантовый эффект Холла.

3. Пространственный профиль электрохимического потенциала

Приведем соображения в пользу формулы (10), (11) для холловского сопротивления, опираясь на общую теорию баллистического транспорта [21].

В модели упругого резистора скорость дрейфа определяется разностью потенциалов

5и = U - U

iНе можете найти то, что вам нужно? Попробуйте сервис подбора литературы.

(16)

между состояниями, связанными со стоком, и состояниями, связанными с истоком, так что вместо уравнения для тока (6) запишем уравнение (126) из работы [21], а именно:

1 (E )=qM (E )|-§1*,

(17)

где для числа мод используется формула (67) из работы [15]

M(E)_ D(E)v(E)

nL

(18)

Из уравнений (102) и (111) работы [21] вместо уравнения (7) имеем разность потенциалов, связанную с напряжением, приложенным к концам проводника,

= qV1 ^ qXV' L + 1 L

(19)

Так же как из (6) и (7) следует формула Друде для проводимости

г I W

& =— = а— , V ь

где удельная проводимость

д'Дм

а = -

WL

(20)

(21)

так и из (17) и (18) следует более общее выражение для проводимости, полученное в [14].

Сейчас покажем, что вместо стандартного уравнения эффекта Холла (8) имеет место уравнение

iH = 2 U в

Vk

W п p

(22)

которое вместе с (17) и (18) дает приведенное выше выражение (10), (11) для холловского сопротивления одномодового упругого резистора.

Ранее в работе [21] было получено уравнение (99)

ди ~dz'

U- Uo

(23)

решения которого определяют электрохимические потенциалы и л (рис. 6).

Рис. 6. Пространственный профиль потенциалов л и Л вдоль длины проводника

Покажем, что эти решения могут быть записаны в виде

и(z,Q~) = и(z)+— Sucosd '

(24)

mc =

v

z

т

Поскольку речь идет об упругом резисторе, электроны в котором движутся при фиксированной энергии и стало быть имеют постоянный импульс, удобно использовать цилиндрические координаты р, в (рис. 7) вместо декартовых рх, ру.

Рис. 7. Ориентация цилиндрических координатр, в относительно канала проводимости

Пусть мы рассматриваем электроны, движущиеся под фиксированным углом в. Тогда с учетом (19) получим

_ дУ

и(г) = и(г) + ^-хтсо$.в , (25)

имея в виду, что в этом простом случае длина свободного пробега

IvT = IvToasO.

(26)

Сравнивая (25) и (24) и выполнив усреднение по углам для 2D проводника (ф-ла (58) из работы [15]), для разности потенциалов получаем дУ п

S/U !

L 2

-VT ,

(27)

и, наконец, комбинируя (25) и (27), получаем искомое выражение (24).

Теперь вопрос в том, как изменится решение (24) уравнения (23) при учете силы Лоренца, создаваемой магнитным полем. Для этого нам потребуется транспортное уравнение Больцмана в приближении времени релаксации [22]:

du du „ du „ du Vx + K + F+

dx dz dpx dp2

(28)

где слева сохранены все х- и г-компоненты, поскольку речь идет о 2D проводнике в плоскости хг (рис. 1). Это уравнение по сравнению с его частным случаем (23) содержит три новых слагаемых. Два из них связаны с силой Лоренца и могут быть переписаны в виде

рдм+рдм=р_у + (29)

x dx z dz u -- r ~

p dO r dr

где радиальная составляющая силы Лоренца (1) отсутствует, а угловая составляющая

^ = 0, ¥в=-дуВ . (30)

Теперь уравнение (28) запишется в виде

у ЁЕ+у Е-Х.ди = и-и (31)

1 дх 2 дг р дв т

С учетом того, что решением уравнения (23) служит уравнение (24), не трудно убедиться, что решением уравнения (31) будет

е(г, в, х) = Е(г) +—дисоъв-—— дВх, (32)

откуда следует

ж p

-дУн =и(х = №)-и(х = 0) = -—ЁЕдВЖ (33)

п р

и соответственно искомое уравнение (22).

Прежде чем перейти к интересующему нас измерению потенциала Холла в четырехтерминаль-ной модели проводника рассмотрим более подробно общие вопросы измерения электрохимических потенциалов.

4. Измерение электрохимических потенциалов. Формулы Ландауэра и Бюттекера

Классическое распределение Ферми (ур-е (10) в работе [15]) с высокой точностью пригодно для массивных контактов, которые всегда остаются практически в равновесном состоянии, но это не обязательно справедливо для небольших проводников даже при малой приложенной разности потенциалов. В [21] показано, как важно ввести два от-

+ -

дельных электрохимических потенциала / и / для понимания физического смысла граничного сопротивления как основы обобщенной формулировки закона Ома. Неравновесные электрохимические потенциалы этого типа оказываются весьма полезными и широко используются в практической нано-электронной инженерии, хотя и поныне все еще высказываются сомнения в их концептуальной ценности и даже измеримости.

Далее на простом примере мы рассмотрим особенности неравновесных потенциалов и вопросы их измеримости, что позволит увязать их с представлениями и формулами Ландауэра и Бюттекера [18, 23-27], которые занимают центральное место в теории транспортных процессов современной мезофизики.

Следуя Ландауэру [23-27], рассмотрим баллистический канал с одним локализованным дефектом, пропускающим долю Т падающих на дефект электронов и отражающим долю 1 - Т назад (рис. 8). Мы могли бы следовать логике рассуждений в [21] и получили бы пространственные профили потенциалов и / в канале с дефектом, как это качественно показано на рис. 8, и таким образом оценили бы сопротивление, оказываемое дефектом, как это проделал Ландауэр еще в 1957 году.

Рис. 8. Профиль неравновесных потенциалов в канале с одним дефектом

Преодолеть высказываемые сомнения в реалистичности неравновесных потенциалов могли бы четырехтерминальные эксперименты (рис. 9) с использованием двух дополнительных контактов, проводящих незначительный ток, с тем, чтобы измерить падение напряжения на дефекте.

Рис. 9. Четырехтерминальный эксперимент по измерению проводимости Ш канала с одним дефектом с коэффициентом пропускания Т

Мы покажем, что в случае одинаковых и слабосвязанных пробных контактов четырехтерминаль-ная проводимость

=

(и -и)

= M-

h 1 - T

(34)

где М - число мод проводника, а двухтерминальная проводимость имеет меньшее значение

G2t =

= M^T.

(¿1 U2)/ q h

(35)

так что различие между ними и есть граничное сопротивление (рис. 10)

1 1 h

G21 GAt q2M

(36)

Рис. 10. Демонстрация того факта, что двухтерминальное сопротивление

iНе можете найти то, что вам нужно? Попробуйте сервис подбора литературы.

^ = 1/ 021 можно рассматривать как сопротивление на дефекте, соединенное последовательно с граничными сопротивлениями.

Хотя граничное сопротивление было предсказано и обнаружено Ю. В. Шарвиным у металлических проводников еще в 1965 году [28, 29], его вездесущая роль признана все еще недостаточно широко и сегодня. В первой половине 80-х годов шла дискуссия о смысле и различии двух формул проводимости (34) и (35), когда, наконец, в 1986 г. Имри не идентифицировал различие между ними (36) как граничное сопротивление [30, 31]. При становлении мезофизики уравнение (35) получило широкую известность и было известно как формула Ландауэра, тогда как действительно оригинальная формула Ландауэра (34) была почти забыта. Падение напряжения на дефекте, контактное сопротивление и обе формулы Ландауэра подробно обсуждаются в обзоре Лесовика и Садовского [32].

Одна из проблем в годы становления мезофи-зики состояла в том, что контакты были сильно связаны с основным проводником и вели себя как «дополнительные дефекты», исключить которые было не так просто. Бюттекер [18, 27] с целью интерпретации четырехтерминальных измерений предложил элегантную формулу

/.. =

(%\LGmn (Um -U), (37)

связывающую ток на контакте с номером т с электрохимическими потенциалами на других контактах, где От,п есть проводимость, определяемая через коэффициент прохождения Ттп между контактами т и п. Для двухтерминальной схемы измерений формула Бюттекера упрощается до

2

I =(%)^ (m ~m2)

= -In

(38)

и совпадает с двухтерминальной формулой Ландауэ-ра (35), если проводимость 012 идентифицировать как (д2/й)М.

Если бы мы знали все химпотенциалы лт, могли бы воспользоваться формулой Бюттекера (37) и вычислили бы все токи 1т. Но в четырехтерминаль-ных измерениях (рис. 9) нам не известны электрические потенциалы на контактах 1* и 2*, так что нам не известны химпотенциалы /и* и /и*. Однако, мы знаем токи I* и I* и они должны быть равны нулю (рис. 9). Если же нам известны на каждом контакте либо ¡т, либо 1т, формула Бюттекера позволяет посчитать все недостающие величины.

Рассмотрим ландауэровское падение напряжения на дефекте (рис. 8) и покажем, что для слабосвязанных контактов формула Бюттекера дает те же са-

+

мые результаты, что и неравновесные потенциалы л и л внутри проводника.

Потенциал, измеряемый пробным контактом (рис. 9), есть некое среднее значение потенциалов ¡л+ и л (рис. 11), при этом корректное усреднение определяется конструкцией контактов. Промоделируем такое измерение путем сопоставления прово-

+ - Т+ т-

димостей g и g двум потокам электронов I и Г (рис. 11).

Рис. 11. Эквивалентная схема для подсчета электрохимического потенциала на пробном контакте

Полагая ток на пробном контакте нулевым,

имеем

так что

(М+-Мз0нд ) + Я~(м~-МзОНд ) = 0, (39)

^зонд

ё +1

где весовые коэффициенты

(40)

g ++g

■ = 1 — а.

(41)

Предполагая, что в измерениях используются неинвазивные пробные контакты атомных размеров, можно ожидать, что проводимости будут практически одинаковыми, так что весовой коэффициент а ~ 50 % и пробный контакт меряет среднее арифметическое

(42)

Изонд =(и++Н—) /2 .

Если контакт недостаточно мал и по разному возмущает потоки 1+ и Г, весовой коэффициент а отличен от 50 %. Если к тому же оба пробных контакта 1* и 2* в измерительной схеме (рис. 9) отличаются весовыми коэффициентами, то измеренное сопротивление дефекта окажется иным по сравнению с ландауэровским значением (34).

5. Формулы Ландауэра

Вернемся теперь к задаче нахождения профиля неравновесных потенциалов в канале с дефектом (рис. 8). Сначала сбалансируем выходящие и входящие токи (рис. 12):

I+R= TI+L+{1 — T) I-, IL-=(1 — T ) I+L+ TIR-.

(43)

Рис. 12. Распределение токов слева и справа от дефекта

Затем, следуя формулам (122) и (123) работы [21], от токов перейдем к функциям распределения

f-+ = TfL +(i—T ) f-f—=(1—t ) f+ + Tf-

(44)

а затем к потенциалам

/+ = Т/ +(1 - Т)/ = Т/ +(1 - Т )/2, / =(1 - Т) / + Т/- = (1 - Т) / + Т/. (45)

Упростим выкладки, выбрав потенциал на одном контакте, равным нулю, а на другом, равным единице (рис. 13).

Рис. 13. Профили потенциалов / и / на дефекте, нормированные на суммарную разность потенциалов, равную единице

Нужную разность потенциалов всегда можно учесть путем умножения на и - Е= ^. Тогда уравнения (45) упрощаются до

U+ - U = T (u-u)

и вместе с выражением для тока

/ = qM (u-u)

(46)

(47)

дают стандартную формулу Ландауэра (35).

Для получения первой формулы Ландауэра (34) находим скачок на дефекте либо потенциала либо /-:

е+-е =(1- т)(е-е ),

Е--Е- = (1 -^(е -е). (48)

а затем делим ток на эту разность и получаем формулу (34).

Не забудем, что нас интересует измерение электрохимических потенциалов внутри проводника. Как это соотносится с измерением напряжения неинвазив-ным зондом сканирующего туннельного микроскопа (СТМ)? Если предположить, что такой зонд СТМ измеряет среднее значение электрохимических потенциалов / и то получим график на рис. 14, сравнение которого с рис. 13 и позволяет утверждать, что неин-вазивный зонд СТМ действительно измеряет среднее значение электрохимического потенциала (м++м~)/2.

Рис. 14. Измерение среднего значения электрохимического потенциала Е+ + Е ) I — сканирующим туннельным микроскопом

Что если зонд СТМ измеряет средневзвешенное значение / и / с весовым коэффициентом а (40), отличным от 50 %? До тех пор пока весовой коэффициент а остается одинаковым для обоих потенциалов / и скачок на дефекте (48) остается прежним

ur-u7d =(i - t )(uu -u),

(49)

что ведет к той же самой первой формуле Ландауэра (34). Если же весовые коэффициенты различны для / и то формула (34) окажется неверной. В предельном случае, когда весовой коэффициент а равен нулю слева и единице справа (рис. 11),

W"-мГ =(1 - 2T )(u-u )■

(50)

что ведет к отрицательному сопротивлению при Т>0.5.

Концепция неравновесных потенциалов / и ведет к интуитивно правильным результатам и сейчас уже широко используется, однако, ее применение требует осторожности. Именно дефект вызывает скачок потенциала, а не область баллистического транспорта (рис. 14), подразумевая, что сопротивление по (34) определяется дефектом. Нужно помнить, что мы все еще рассматриваем упругие резисторы. Да, скачок напряжения 1Я остается, а диссипация джоулева тепла 12Я в проводнике отсутствует и происходит лишь на контактах.

6. Формула Бюттекера

Его формула (37) содержит непосредственно измеряемые величины, оставляя за ее пределами любые вопросы, касающиеся немых внутренних переменных. Хотелось бы подчеркнуть общую применимость формулы Бюттекера, независимо от того проводник является упругим или не является таковым. Действительно, как мы сейчас увидим, для вывода формулы (37) потребуется лишь совсем немного сверх обычной линейной теории электрических цепей.

Определим для начала мультитерминальную проводимость

di„

d(u 1 q)'

G = I dIm m,m " d(um I q) '

m Ф n

(51)

где естественно задаться вопросом, почему при т Ф п знак выбран отрицательным в отличие от случая, когда т=п. Выбор знака можно проиллюстрировать на примере четырехтерминального измерения канала с одним дефектом (рис. 15).

Увеличение химпотенциала /] ведет к увеличению тока, входящего в контакт 1 (положительный ток), и к оттоку от всех других контактов (отрицательные токи). Знаки в (51) выбраны таким образом, чтобы коэффициенты проводимости Gщn всегда были положительными, как того интуитивно требует смысл понятия проводимости.

Рис. 15. Мысленный эксперимент с четырехтерминальным измерением канала с одним дефектом как на рис. 8

Теперь ток (37) перепишем через коэффициенты проводимости

Е

I„ = G„

-У G

/ m

(52)

при этом коэффициенты должны удовлетворять следующим двум правилам суммируемости.

Во-первых, если все химпотенциалы одинаковы (состояние равновесия), все токи по (52) должны быть равны нулю, что влечет за собой правило сумм

G = У G .

m,m / t m,n

(53)

Во-вторых, при произвольных значениях хим-потенциалов сумма всех токов должна быть равна нулю, что приводит ко второму правилу сумм

G = У G

m, m / , n,

доказательство которого не столь очевидно, как это имело место с первым правилом сумм, и получить его можно опираясь на уравнение (52). Сначала просуммируем все токи (52)

У Im = 0 = У G^ ^ ^У Gt

u

m,r.

m q

m,

m n^m q

(54)

iНе можете найти то, что вам нужно? Попробуйте сервис подбора литературы.

затем воспользуемся правилами суммирования (53) и поменяем местами индексы в двойной сумме

о = У G u-УУ G u,

/ j m m / * / j n,m

m q m n^m q

(55)

что при произвольных значениях химпотенциалов справедливо только, если удовлетворяется второе правило сумм (53). Окончательно,

G =У G =У G .

m.m / ^ m.n / j n,m

(56)

Воспользовавшись правилами сумм (56), перепишем первое слагаемое в (52) в виде суммы и в результате получим знаменитую формулу Бюттикера (37). Обратим внимание на то, что в формуле Бютти-кера суммирование нет нужды ограничивать по пфт, поскольку слагаемое при п=т всегда равно нулю.

Иногда формулу Бюттекера (37) удобнее записать через так называемые коэффициенты отклика [33]

Im =У i

Ип_

q

(57)

которые определяются следующим образом:

8т,п п , т * П

8 = G

с>т,т т,т

с правилами суммирования

У я = У Я = 0 .

/ 1 о т п / ' о п т

(58)

(59)

Применим подход Бюттекера к ситуации на рис. 14, где утверждается, что два неинвазивных зонда, расположенные до и после дефекта с коэффициентом пропускания Т, генерируют потенциалы соответственно 1 - Т/2 и Т/2 (рис. 16).

Рис. 16. Генерирование на зондах потенциалов 1 - Т12 и Т12 соответственно до и после дефекта с коэффициентом пропускания Т

Получим ли мы те же результаты методом Бюттекера? Имея в виду, что в этом измерении речь идет о четырех токах и четырех потенциалах на контактах 1, 2, 1* и 2*, запишем уравнение (57) в матричном виде

11

12

11*

I2*

Mq

A C

u u u*

u2*

(60)

где матрицы А, В, С и Б имеют форму (2 х 2). Поскольку

то можно записать, что

Е* Е*

= -D-C

с матрицами С и D в виде

~-ti -t2

[C D] =

-t'2 -t[ 0

(61)

(62)

(63)

где элементы матрицы С интерпретируются как вероятности межконтактного транспорта 1 на 1*, 2 на 1*, 2 на 2* и 1 на 2* (рис. 17), причем предполагается, что контакты 1* и 2* слабо связаны и транспортом между ними можно пренебречь.

n

m

Рис. 17. Вероятности межконтактного транспорта при четырехтерминальном измерении проводимости канала с одним дефектом с коэффициентом пропускания Т

Правила сумм (59) дают

r = ^ +12,

г' = ^ + г2'. Из уравнений (62)-(64) имеем

(64)

U* =

—^ U +— ^ + t2 t +1

" U2,

(65)

¿2*

и +-

¿2.

t +12

t +12

Учтем также, что вероятность ^ (рис. 17) может быть представлена как сумма вероятности т прямого переноса 1^-1* и вероятности (1-Т)т отразиться от дефекта, а затем достичь контакта 1*, и аналогичные соображения для вероятности 12:

^ = т + (1 - Т)т

h = Тт,

(66)

поскольку на пути электрона с контакта 2 сначала нужно преодолеть дефект (вероятность Т), а затем попасть на контакт 1* (вероятность г). По тем же соображениям ^ = ^ и /2 = t2 (рис. 17).

Пользуясь уравнениями (66) и положив ц1=1 и ¡2=0, из уравнений (65) получаем ожидаемые значения для потенциалов на зондах 1* и 2*:

U* = 1 -(Т/2), U2* = Т / 2,

(67)

в чем можно было бы и не сомневаться, поскольку в формуле Бюттекера фигурируют только характеристики контактов и фактически свернуты все тонкости, связанные с неравновесными электрохимическими потенциалами.

6. 1. Не тривиально ли уравнение Бюттекера?

В конце концов, любую сложную электрическую схему всегда можно представить эквивалентной схемой эффективных резисторов, как например на рис. 18 для трех терминалов.

Даст ли нам стандартная теория электрических цепей такой же результат как и уравнение Бюттекера (37)? Ответ на этот вопрос несомненно положительный, однако, только для нормальных взаимных электрических цепей, у которых прово-

димости между всеми терминалами полностью обратимы:

SJ _ SJ

m^n n^m >

(68)

другими словами, электроны одинаково легко транспортируются в обе стороны для каждой пары терминалов (т, п).

Рис. 18. Эквивалентная схема эффективных резисторов для трех терминалов

Область применимости же уравнения Бютте-кера (37) намного более широкая, чем нормальные взаимные электрические цепи, например, она применима для проводников в магнитном поле, когда не имеет места взаимная проводимость:

G Ф G

m^n n^m

(69)

Для таких ситуаций обосновать применимость уравнения Бюттекера (37) не так то просто. Если в уравнении Бюттекера (37) проводимость Gщn поменять на Gn,m, то мы просто получим ошибочную формулу, применяя которую к многотерминальным невзаимным цепям получим ошибочные результаты, не соответствующие формуле Бюттекера (37).

7. Измерение потенциала Холла

Обратимся, наконец, к вычислению холлов-ского напряжения с использованием формулы Бют-текера (37) при четырехтерминальном эксперименте, отличающемся от рассмотренного выше (рис. 17) размещением пробных зондов 1* и 2* поперек проводника (рис. 19).

Рис. 19. Вероятности межконтактного транспорта при четырехтерминальном измерении холловского напряжения

В отсутствии магнитного поля разность зондо-вых потенциалов нулевая. При включении магнитно-

го поля перпендикулярно к поверхности проводника (плоскости рисунка) электроны, движущиеся слева направо, отклоняются вдоль или против направления движения к боковым краям проводника (рис. 1), создавая различие между зондовыми потенциалами * и л2* , которое мы и хотим вычислить в зависимости от напряжения, создающего ток в проводнике.

Уравнения для зондовых потенциалов остаются прежними (65), если вероятностям межконтактного транспорта придать смысл, показанный на рис. 19. Вычисление этих вероятностей либо численно, либо в полуклассическом приближении приводит к хол-ловскому напряжению VH

—qVH = и* — L*

(70)

в согласии с уже полученным ранее уравнением (22).

Уравнение (22) было получено опираясь на транспортное уравнение Больцмана, но его можно получить и из уравнений (65), если переписать их применительно к тонкому проводнику ширины где-нибудь посередине длинного проводника меньшей,

чем средняя длина свободного пробега, с электрона-

+

ми, движущимися слева, где потенциал л , и справа, где потенциал л, для потенциалов на середине проводника на его боковых сторонах в точках, имитирующих зонды 1* и 2* (рис. 20), а именно:

iНе можете найти то, что вам нужно? Попробуйте сервис подбора литературы.

и( x + dx) =---¡U н—-— и

v ; t + t2 К +t2

v(x) = ——-l++----U .

(71)

К + h

К + К2

Рис. 20. Графическое пояснение к выводу уравнения (22) из вероятностей межконтактного транспорта при четырехтерминальном измерении холловского напряжения

Для вычисления вероятностей в уравнениях (71) заметим, что в отсутствии магнитного поля электроны со скоростями, лежащими в пределах угла в (рис. 5) 0 <в <+ж/2 движутся вверх, а лежащими в пределах -ж/2 < в < 0 движутся вниз (рис. 21).

0<в<+п/2,

движутся вверх, а лежащими в пределах

-л/2<6><0, движутся вниз (рис. 21).

0=-тс/2

Рис. 21. Графическая демонстрация вычисления угла заворота электрона под действием силы Лоренца

Магнитное поле закручивает траекторию электронов наверх на угол, равный произведению угловой скорости (14) на время dx/v, затрачиваемое на преодоление расстояния dx

qvß dx qB dx

(72)

Тогла вероятности межконтактного транспорта будут пропорциональны углам

К = К1

t2 = t2

п qB , • — + — dx, 2 р

п qB ,

----dx.

(73)

2 р

и подстановка их в уравнения (71) дает

U(x + dx) —v(x) = 2qBdx (¡S — и ), (74)

так что

Tip

du 2qB , -J— = ——ou dx np

(75)

в согласии с уравнением (32), а стало быть и с (22).

8. Квантовый эффект Холла, уровни Ландау и краевые состояния в графеме

Одним из наиболее эффектных приложений подхода Бюттекера служит интерпретация режима квантового эффекта Холла, когда магнитное поле настолько велико, что электроны, движущиеся от истока, не достигают зонда 2*, а электроны, движущиеся от стока, не достигают зонда 1* (рис. 19). В результате

Vh

Л — К2

(u —¡2 )/ q t, +12

= i,

(76)

поскольку t2=t2 '=0, так что холловское напряжение оказывается равным разности потенциалов на кон-

цах проводника, а холловское сопротивление равным обычному двухтерминальному сопротивлению.

Интересно, что в этом режиме сопротивление дается выражением (4) с высочайшей степенью точности [4]. Это можно представить себе так, как будто бы мы имеем идеальный баллистический проводник с только граничным сопротивлением. Длина проводников в экспериментах нередко достигает нескольких сот микрометров, подобная баллистичность просто удивительна, что и было отмечено Нобелевской премией в 1985 году (Клаус фон Клитцинг).

Обратимся к графену. В предыдущей публикации [19] сформулирован метод неравновесных функций Грина (НРФГ) в матричной формулировке применительно к решению модельных транспортных задач наноэлектроники, в частности, для графеновых лент были приведены результаты расчета методом НРФГ коэффициента прохождения и плотности состояний для двух конфигураций ленточных границ -зигзагообразной и креслоподобной в приближении сильной связи в ортогональном базисе с параметрическим учетом взаимодействия лишь соседних атомов через кулоновский е и резонансный ^ интегралы (хюккелевское приближение) в полном согласии с результатами вычислений с более точными модельными гамильтонианами. Сейчас мы учтем внешнее магнитное поле для графеновых лент в этом же приближении и приведем результаты расчета коэффициента прохождения и плотности состояний в присутствии магнитного поля, а также квантового эффекта Холла и проводимости графеновых лент в магнитном поле [34].

Изучаемая нами система представляет собой длинный 2D проводник с двумя пробными зондами по обеим сторонам проводника (рис. 22) для измерения поперечного холловского напряжения, возникающего в присутствии магнитного поля, на -правленного перпендикулярно к поверхности 2D проводника.

Рис. 22. Измерение холловского напряжения через разность химпотенциалов, создаваемую между поперечными зондами при включении перпендикулярного к проводнику магнитного поля

Вначале учтем магнитное поле в гамильтониане проводника Н и контактных матрицах и Е2 метода НРФГ.

Энергия частицы с зарядом -д в поле векторного потенциала А дается выражением

Е(г,р)=(Р + дЛ^ + дЛКи (Г). (77)

Для нахождения параметров хюккелевского гамильтониана с учетом магнитного поля рассмотрим однородную решетку атомов с постоянным значением Ес и постоянным векторным потенциалом. Сначала рассмотрим Ш проводник, вытянутый вдоль оси х

Е(р) = Е +(рх + дАх)(рх + дАх) , (78) так что дисперсионное соотношение имеет вид

+ (79)

и может быть аппроксимировано функцией косинуса

(80)

с хюккелевскими параметрами е и t [19]

Е (кх) = е + 2t cos +

£ = Ec-2t, t = -fr / 2mcr = —L

(81)

где а - постоянная решетки. Отличие от однородной решетки без учета поля векторного потенциала [19] состоит лишь в появлении фазового множителя с\р(±/<у/1,й/7/) у резонансных интегралов (рис. 23).

Рис. 23. Однородный Ш проводник в поле векторного потенциала

Для однородной решетки 2D проводника фазовые множители с фазами

qA a qAya

ti

b

(82)

появляются у резонансных интегралов в обоих направлениях плоскости проводника (рис. 24). Магнитное поле

В=УхА.

(83)

так что интересующее нас магнитное поле, перпендикулярное плоскости 2D проводника

в =dk_ A

z dx dy

Рис. 24. Графическая демонстрация того, что учет магнитного поля, направленного вдоль оси г, сводится к учету фазовых множителей у резонансных интегралов хюккелевской решетки

Холловское сопротивление дается отношением холловского напряжения к току по проводнику (9). Холловское напряжение можно вычислить двумя способами. Используя четырехтерминаль-ную измерительную схему и уравнение Бюттекера, вычислить разность химпотенциалов, создаваемых на двух поперечных зондах (рис. 22). По другому, в рамках метода НРФГ [19] след неравновесной функции Грина о"

2nN = Tr [ Gn ]

(85)

пропорционален числу электронов, а спектральная функция А

2nD (E ) = A

(86)

пропорциональна плотности состояний, так что заселенность состояния ] есть отношение соответствующих диагональный элементов функции Грина и спектральной функции

f (j ) =

Gn (j, j) A(j,j)

(87)

В области линейного отклика заселенность пропорциональна химпотенциалу [15]. Если на одном контакте положить А=0, а на другом А=1, так что разность между ними есть qV, то заселенность А/), вычисленная по (87), будучи умноженной на qV, даст значение химпотенциала

и( j ) = qVf ( j ).

(88)

На рис. 25 показаны результаты вычисления холловского сопротивления, нормированного на квант сопротивления И^2, как функции магнитно -го поля.

Замечательный результат - это появление ступенек в сильных магнитных полях, известный как квантовый эффект Холла. Однако, сначала обратимся

к области слабых магнитных полей, где холловское сопротивление, согласно стандартной полуэмпирической теории эффекта Холла, зависит от магнитного поля линейно (9)

R = B / qn,

(89)

что и показано на рис. 25 пунктирной линией, вычисленной для

n = -

N _ k2 LW ~ 4п

(90)

при E = t0 = 2.7эВ и ka = п/3 с a = 2.5нм [19]'

Рис. 25. Схема измерения холловского напряжения V, и нормированное холловское сопротивление Л

в зависимости от магнитного поля Б для графеновой ленты шириной W = 26а = 65 нм в хюккелевском

приближении при энергии Е = ^ [19]

С ростом магнитного поля постепенно возникает ступенчатая зависимость холловского сопротивления от поля, обязанная квантовым эффектам в виде уровней Ландау (рис. 26). На этом экранном снимке отложены диагональные элементы спектральной функции А(у,у;Е) в зависимости от энергии Е,

вычисленные методом НРФГ, и представляющие собой локальную по ширине ] графеновой ленты плотность состояний для трех уровней Ландау.

Энергия уровней Ландау

iНе можете найти то, что вам нужно? Попробуйте сервис подбора литературы.

Е„ = | и +

(91)

где п - целое число, юс - циклотронная частота (15). Эквидистантные уровни Ландау располагаются друг относительно друга с инкрементом

fiqB 2qBa2 tmc =-<- = -<—10 m n

(92)

выражение для которого получено с использованием (15) и (81). При использованных в расчете значениях 5=20 Т и ¡7=2.5 им инкремент получается равным /?« ~ 0.37/0 в неплохом согласии с результатами расчета методом НРФГ (рис. 26).

Краевые состояния

L J

Рис. 26. Локальная по ширине графеновой ленты плотность состояний в зависимости от энергии уровней Ландау при В=20 Т

Выражение для энергии уровней Ландау (91) есть чисто квантовый результат аналитического решения уравнения Шредингера с учетом векторного потенциала, лежащего также в основе численного решения методом НРФГ для конкретной геометрии графеновой ленты. Этот результат можно понять качественно, если обратиться к полуклассической модели, согласно которой электроны движутся по круговым орбитам, совершая один оборот за время

tc =

2л 2л p

юс qvB

так что длина окружности орбиты с радиусом rc

2л p

2л г = vt = -

qB

(93)

(94)

Если теперь учесть квантовое ограничение, согласно которому длина окружности орбиты должна быть такой, чтобы на ней укладывалось целое число К де-бройлевских длин волн к /р , то получим

2л p qB

= K ■ h / p .

(95)

Полуклассически электрон может иметь любую энергию Е = р2/2ш, однако, квантовое ограничение в виде равенства длины окружности целому числу К длин волн ведет к

p2 =K ■ hqB, так что разрешенные значения энергии

(96)

Е = КГирВ_ = кТш1_ 2m 2

(97)

что не совсем то же самое, что и правильный результат (91), но качественно вполне приемлемо.

Если число краевых состояний обозначить К , то результирующий ток

I =—V ■ K

h es

(98)

поскольку холловское напряжение Ун просто равно напряжению V , приложенному к концам проводника, так как один край проводника находится в равновесии с истоком, а другой - со стоком.

Это ведет к квантованию холловского сопротивления

Rh =

h _L

7 K

(99)

порождая у нормированного холловского сопротивления плато при 1А, Уъ, У и 1 (рис. 25) по мере того как магнитное поле генерирует уровни Ландау, изменяя число краевых состояний Kes при энергии E = t0 c 4-х до 3-х, до 2-х и до 1-го (рис. 26).

Что касается распределения электрического тока в 2D проводнике в режиме наблюдения квантового эффекта Холла, то часто используют [35, 36] полуклассическую модель движения электронов по так называемым [37] открытым орбитам (skipping orbits) (рис. 27). В результате образуются краевые токи, обеспечивающие исключительно высокую баллистическую проводимость.

S D

Мг 1

Рис. 27. Графическая демонстрация того как электроны в 2D проводнике в сильном магнитном поле движутся по открытым орбитам, создавая компактные трассы краевых токов, как бы локализованных в квантовом волноводе [37]

Ранее уже приводились результаты расчета методом НРФГ коэффициента прохождения и плотности состояний графеновых лент для двух конфигураций ленточных границ - зигзагообразной ZGNR / Zigzag Graphene NanoRibbon и креслоподобной AGNR / Armchair GNR в отсутствии магнитного поля [19]. Влияние сильного магнитного поля показано на рис. 28.

Уровни Ландау LL/Landau Levels в сильных магнитных полях образуются в таких 2D проводниках, ширина которых достаточна для образования циклотронных орбит. Чем больше ширина проводника, тем требуется меньшее магнитное поле для генерации первого LL. В графене энергии уровней Ландау

£f =±i'0>/2 qtmB,

(100)

где фермиевская скорость принималась равной у0= = 106 м/с [38]. Согласно (100), первый и второй уровни Ландау при В = 20 Т возникают при Е^ = 0.15 эв и Е^ = 0.23 эв. Есть также уровень Ландау при Е = 0, это нулевой уровень Ландау Е^ [39-41]. Результаты расчета методом НРФГ (рис. 28) полностью согласуются с приведенными оценками.

Рис. 28. Влияние сильного магнитного поля в = 20Т на коэффициент прохождения Т(Е) и плотность состояний Б(Е) графеновых лент шириной Ж = 53 нм для двух конфигураций границ -ZGNR и АGNR [34]

9. Формула Кубо

Точно так же как тепло передается от более нагретого конца проводника к менее нагретому, а различие в уровнях жидкости ведет к её перетоку, так и электрический ток течет от контакта с большим значением электрохимического потенциала к контакту с меньшим его значением [14]. Под направлением тока, вопреки исторически сложившейся договоренности, мы в этой серии статей понимаем направление движения электронов. Неоднократно подчеркивалась неравновесная природа образования электрического тока [19]. Общее выражение для тока [14]

1 = ^^(Е)"л (101)

справедливо для любой по величине разности потенциалов, подаваемой на концы проводника. В этом выражении Б (Е) - плотность состояний в проводнике, t (Е) - время пролета электрона от истока до стока Б, ^ (Е) - фермиевские функции контактов (/ = 1,2). В случае интересующего нас низковольтного предела проводимость [14]

о=✓ f * [-§ 1»<E >

дБ ) 21 (E )

(102)

Хотя это выражение и было получено из общего неравновесного выражения (101), полезно отметить, что проводимость в режиме линейного отклика является в действительности равновесным свой-

ством. Имеет место фундаментальная флуктуацион-но-диссипационная теорема, связывающая проводимость в квазиравновесном режиме с токовими флук-туациями в состоянии равновесия, когда разность потенциалов на концах проводника нулевая.

При ненулевой температуре тепловые флуктуации являются одной из причин зашумленности проводника с током, что приводит, в частности, к флуктуации заселенности состояний проводника. Заселенность состояний проводника g обычно характеризуется числом, которое может принимать значение 1 или 0. Термодинамически среднее значение заселенности определяется функцией Ферми

f (E) = 1/[e'

(ß-ц)/kT

1 Ч ^ •

В равновесном состоянии вероятность того, что состояние не заселено, дается в среднем значением 1 - /, а вероятность того, что состояние заселено в среднем равно / . Флуктуации заселенности от их средних значений характеризуются выражением

(ё-<*))2 = ё2 -2g{g) + <¿)2.

Для фермиевских систем ё2 = ё, так что флуктуации заселенности (в состоянии равновесия) от их термодинамически средних значений даются выражением

((g -< g> )2)=f (1 - f) •

Эти среднеквадратичные флуктуации исчезают в нулевом температурном пределе. При высоких температурах и достаточно больших энергиях фер-миевская функция намного меньше единицы, так что множитель 1 - / можно заменить на 1: флуктуации подчиняются статистике Максвелла - Больцмана.

Флуктуации заселенности порождают токовые флуктуации (в состоянии равновесия) во внешней цепи, которые через флуктуационно-диссипаци-онную теорему, иначе формулу Кубо, связаны с проводимостью проводника. Таким образом, исследование токових флуктуаций в состоянии равновесия дает нам ту же самую информацию, что и исследование проводимости. Обсудим этот вопрос подробнее.

Рассмотрим ситуацию, когда проводник не находится под напряжением, электрохимический потенциал на контактах 5 и Б один и тот же ¿и0 (рис. 29).

Рис. 29. Графическая демонстрация того, что в состоянии равновесия когда электрохимический потенциал один и тот же ¿и0 на обоих контактах, суммарный ток нулевой, однако, имеют место взаимно компенсирующиеся токи I между контактами

Время от времени некоторое число электронов проходит по проводнику от к Б и такое же число электронов возвращается от Б к 5 так, что

i1('<>)> „ = 0,

(103)

где скобки означают либо усреднение по ансамблю многих одинаковых проводников, либо усреднение по времени в течение времени пролета /0.

Однако, вычисления показывают, что токовый коррелятор

C, = J dr(l(t0 +t)I(t0))a

(104)

отличен от нуля даже в состоянии равновесия, и формула Кубо увязывает этот коррелятор с проводимостью в режиме линейного отклика [42, 43]:

G =-C

1

2kT 2kT

Jdr{I(t0 +r)I(t0))q . (105)

Формула Кубо позволяет найти проводимость путем вычисления токовых корреляций методами равновесной статистической механики, которые разработаны намного лучше по сравнению с методами неравновесной статистической механики. Это действительно сильный результат. Еще до триумфального появления мезоскопической физики в конце 80-х годов формула Кубо была фактически единственным подходом в моделировании квантового транспорта в электронике. Единственно, ее применение ограничено транспортными задачами в режиме линейного отклика. Позже был окончательно сформулирован метод неравновесных функций Грина [19], который стал основным инструментом рассмотрения неравновесных задач квантового транспорта так же как транспортное уравнение Больцмана является основой для решения транспортных задач в полуклассической формулировке [44].

Формула Кубо применима в принципе к массивным проводникам в диффузионном режиме. Однако, на практике трудно оценить влияние сложных неупругих процессов на токовые корреляции. Обычно рассматривают проводники в высокочастотном поле, в котором электроны дергаются туда и сюда так и не достигая истока и стока. Оказывается, однако, возможным найти проводимость при нулевой частоте [45].

Мы рассмотрим далее простую задачу о влиянии контактов на токовые корреляции в упругом резисторе. Покажем, что применительно к упругому резистору формула Кубо приводит к известной формуле (102). Затем конспективно обсудим соотношения Онзагера.

9. 1. Формула Кубо для упругого резистора.

Как и ранее в концепции «снизу - вверх» [14], рассмотрим одноуровневый резистор с одним и тем же электрохимическим потенциалом /л0, а стало быть и с одной и той же фермиевской функцией /0 (Е) на контактах (рис. 30).

Рис. 30. Графическая иллюстрация того, что при

равновесии (с одинаковым значением электрохимического потенциала ¿и0 на контактах) тока нет, однако, имеют место случайные импульсы тока при проскакивании электрона от одного контакта к другому и наоборот

Происходят случайные положительные и отрицательные импульсы тока, вызванные проскакива-нием электрона от 5 к Б и наоборот, соответственно. Положительный ток в среднем одинаков с отрицательным током, и этот равновесный ток /0 можно выразить через время пролета (ф-ла (24) в [14]), а именно:

i0 = qf0(S) [1 - f0(S)]

(106)

где произведение /0 (е)[1 - /0 (е)] есть вероятность

того, что на контакте 5 есть электрон, готовый к переходу на Б, а на контакте Б нет электрона, готового к переходу на 5. Корреляция электронных переходов имеет место, поскольку предполагается, что переходы носят стохастический характер.

Подынтегральное выражение коррелятора (104) выглядит как последовательность треугольных импульсов (рис. 30) каждый площадью д2//, так что

2q

C = 2^f0(s)[l - f0(s)],

(107)

где появление двойки вызвано тем, что ток (106) учитывает только положительные импульсы в то время как вклад в коррелятор дают также и отрицательные импульсы.

iНе можете найти то, что вам нужно? Попробуйте сервис подбора литературы.

Сейчас покажем, что формула Кубо (105) применительно к упругому резистору дает известное выражение (102) для проводимости. Обобщая модель одноуровневого упругого резистора на таковой с произвольной плотностью состояний, имеем:

7 D (E) I0 = q J dE-±-L q-J 21 (E)

f0(E)[1 - f0(E)], (108)

С, = 2q2 jdEDj-E)f0(E)[1 -f0(E)], (109) -t 2t (E)

так что

ci = 2qio • Согласно же формуле Кубо (105),

г*

G = = q2 f dE

7lrT f

2 kT

D (E) fo(E) [1 - fo(E)]

21 ( E ) kT

(110)

(111)

что сразу дает искомое выражение для проводимости (102), поскольку [14]

fo( E)[1 - fo(E)]

kT

dE

(112)

Равенство (112) играет ключевую роль в выводе выражения для проводимости (102). Ведь элементарные соображения показывают, что проводимость пропорциональна [14]

¡лх -ß2 dE '

(113)

тогда как учет токовых корреляций по формуле Кубо приводит к проводимости, пропорциональной

/0(Е) [1- /0(Е)] /кТ .

Итак, изучение токових флуктуаций (в состоянии равновесия) эквивалентно исследованию проводимости в режиме линейного отклика. Это не так в случае дробового шума в электрических цепях, обусловленного дискретностью заряда электронов как носителей электрического тока. Подобные тонкие корреляционные эффекты в упругих резисторах в связи с развитием мезоэлектроники интенсивно изучаются [46].

Рис. 31. Графическая иллюстрация того случая, когда магнитное поле ориентировано таким образом, что электрон с контакта 2 закручивается на контакт 1, а электрон с контакта 1 закручивается от контакта 2, так что невзаимность проводимостей О 2 Ф О21 воможна, если число терминалов больше двух

Однако, для трехтерминального устройства уже имеем

G1,1 = G1,2 + G1,3 = G2,1 + G3,1 :

(116)

и таким образом снимается требование взаимности проводимости. Эффекты такой невзаимной проводимости с очевидностью обнаружены на примере краевых состояний в режиме квантового эффекта Холла, к чему мы еще вернемся ниже.

Концепция краевых состояний, обеспечивающих однонаправленный баллистический транспорт на макроскопические расстояния, представляется исключительно интересной несмотря на то, что наблюдение таких состояний ограничено низкими температурами и сильными магнитными полями, что малоприемлемо с практической точки зрения. Ситуация, однако, может измениться с открытием новых материалов, например, таких как топологические диэлектрики, которые демонстрируют краевые состояния даже без приложения магнитного поля.

9. 2. Невзаимные электрические цепи.

В электрических цепях с взаимной проводимостью имеет место правило сумм (56):

О =У О =У О (114)

т т / , т п / , п,т V '

пфт пфт

Обратим внимание на любопытное и полезное свойство цепей с невзаимной проводимостью с участием, например, проводников с краевыми токами, обеспечивающих исключительно высокую баллистическую проводимость, электроны в которых движутся по так называемым открытым орбитам [37].

Рассмотрим двухтерминальное устройство (рис. 31) с такой ориентацией магнитного поля, что электрон с контакта 2 закручивается на контакт 1, а электрон с контакта 1 закручивается в сторону от контакта 2.

Для двухтерминального устройства невзаимность проводимостей О 2 Ф О21 невозможна, поскольку согласно правилу сумм (114)

Оц = 01,2 = 02Л . (115)

9. 3. Соотношения Онзагера

Зададимся вопросом, возможна ли невзаимная проводимость в отсутствии магнитного поля. В общем случае матрица проводимости подчиняется соотношениям взаимности Онзагера

О, (+В) = От, (-В), (117)

согласно которым ток на терминале п, обязанный напряжению, поданному на терминал т, равен току на терминале т, обязанному напряжению, поданному на терминал п, с одновременным обращением направления магнитного поля на противоположное. Это один из важнейших результатов статистической механики. Мы докажем его ниже опираясь на формулу Кубо для многотерминальных цепей.

Сейчас же зададимся вопросом, требуют ли соотношения Онзагера (117) взаимной проводимости в отсутствии магнитного поля (В=0). Ответ положительный, но при условии, что проводники изготовлены из немагнитных материалов. В противном случае, взаимность обеспечивается не только обращением магнитного поля, но и обращением знака намагниченности материала:

Gn, (+B, +M) = Gm, (-B, -M).

(118)

Например, пусть один из контактов изготовлен из магнитного материала (рис. 32).

Рис. 32. Графическая демонстрация равенства (118): (а) контакт 1 имеет отличную от нуля намагниченность; (б) взаимность цепи 1-2 при обращении магнитного поля требует изменения знака намагниченности контакта 1

Тогда, согласно соотношениям Онзагера (118), проводимость 012 цепи (а) должна совпадать с проводимостью 02 1 цепи (б). Однако, для одной и той же цепи, (а) или (б), проводимости 2 и 1 не совпадают.

При нынешнем понимании физики топологических диэлектриков, являющихся немагнитными материалами, топологические диэлектрики не обнаруживают невзаимную проводимость с немагнитными контактами в отсутствии магнитного поля, однако могут проявить невзаимность с магнитными контактами. Эти и ряд других аспектов физики топологических диэлектриков еще ждут своего более глубокого понимания [47].

Важнейшим приложением формулы Кубо (105) является доказательство справедливости соотношений взаимности Онзагера (117). Для этого используется многотерминальная версия формулы Кубо

1

— J dz(lm(t0 +т)1п(ta))eq, (119)

G = m,n 2kT

учитывающая корреляцию токов на двух различных терминалах.

Рассмотрим трехтерминальную цепь с магнитным полем (В > 0), которое электроны, входящие в контакт 1, закручивает в направлении контакта 2, те же электроны, которые входят в контакт 2, закручивает в направлении контакта 3, и электроны, входящие в контакт 3, закручивает в направлении контакта 1 (рис. 33).

Можно ожидать, что коррелятор (/2 (/0 +т)/\ (¡0)) должен выглядеть приблизительно

как на рис. 34 с корреляцией, спадающей медленнее с ростом т > 0.

Объясняется это тем, что электроны движутся от первого терминала ко второму (рис. 33), и таким образом ток / в момент времени сильнее скоррелирован с током / в последующие моменты времени (т > 0), чем с током в предыдущие моменты (т< 0). Обращение направления магнитно-

го поля на противоположное (В < 0) изменяет траектории движения электронов на обратные, так что

+ТУ2(0)ед В <0= Ы Т^)),, В >0 . (120)

Считая это утверждение справедливым, соотношения Онзагера (117) немедленно следуют из формулы Кубо (119).

Рис. 33. Трехтерминальное устройство с магнитным полем, направляющим электроны с одного терминала на другой

Рис. 34. Токовый коррелятор в окрестности момента времени при движении электронов от терминала 1 к терминалу 2 (качественно)

Соотношения Онзагера, рассмотренные выше, являют собой лишь частный случай общих соотношений Онзагера для обобщенных транспортных коэффициентов разного вида [43].

Вернемся, однако, к спиновым эффектам и их учету в методе НРФГ.

10. Спинорное представление

Применение магнитных контактов для генерирования спинового тока в немагнитном канале проводимости даже за пределами протекания тока с использованием пробного магнитного контакта 2 с поляризацией Р2 (рис. 35) рассматривалось в [21] на основе концепции о зарядовом химпотенциале и и спиновом потенциале .

Связь между зарядовым и спиновым потенциалами дается простым выражением

/¿2 =М +

(121)

2

в котором вектор поляризации совпадает с направлением регистрирующего контакта 2 относительно направления тока, а вектор спинового потенциала совпадает с направлением спиновой поляризации канала проводимости, которое по договоренности есть направление намагниченности инжектирующего контакта. Обычно рассматриваемые параллельная и антипараллельная ориентации намагниченности контактов являються частными случаями общего выражения (121). У нас пока нет надежных сведений, выполнялись ли на самом деле подобные эксперименты, т. е. спины инжектировались контактом с фиксированным направлением намагниченности, а разность потенциалов замерялась вращающимся магнитным контактом. Неоднократно, однако, експериментально измерялась фактически аналогичная ситуация, когда оба магнитных контакта были фиксированы друг относительно друга, а спин электронов в проводнике вращался внешним магнитным полем.

Рис. 35. Регистрирующий контакт 2 в роли анализатора спиновото тока

Все подобные эффекты исключительно интересны и важны сами по себе. Мы же, однако, далее займемся тем, что покажем как подобные спиновые эффекты учесть в методе НРФГ [19] применительно к модельным транспортным задачам наноэлектрони-ки, в частности, к рассмотрению квантового спинового эффекта Холла в графене. Покажем также как метод НРФГ приводит к результатам, соответствующим уравнению (121). Учет спина в методе НРФГ потребует введения матричных потенциалов формы (2 х 2), которые, как окажется, могут быть выражены через зарядовые и спиновые потенциалы, фигурирующие в (121).

Спин электронов и поляризация фотонов во многом похожи, за исключением того, что понятие ортогональности для них имеет разный смысл. Для фотонов ортогональность поляризации реализуется при 90°, а для электронов - при 180°, что проявляется в колебании нелокального спинового потенциала в зависимости от угла между инжектирующим и регистрирующим магнитными контактами (рис. 36) [21].

Математически адекватное описание ортогональности электронных спинов требует спинорного исчисления, а не векторного как в случае поляризации фотонов. Вектор описывается тремя действительными компонентами вдоль осей х, у, г декартовой системы координат, а спинор описывается двумя комплексными компонентами вдоль направлений вверх (ир) и вниз (йп)\

Wup Wdn

где здесь и далее, как и прежде [19], векторы-столбцы будем брать в фигурные скобки, а квадратные матрицы - в прямоугольные.

Рис. 36. Колебания нелокального спинового потенциала в зависимости от угла между инжектирующим и регистрирующим магнитными контактами (рис. 35)

И тем не менее, спинор нужно воспринимать как объект, имеющий определенное направление как и вектор. Как же согласуется такое визуальное восприятие спинора с его двухкомпонентной структурой?

Спинор, ориентированный вдоль направления единичного вектора п с проекциями на оси х, у, г

n =

sin в cos $ sin в sin q cose

(122)

описывается двумя комплексными компонентами c и s:

cos- в-'ф/2 = c

2

в — <

2

sin - e+iq/2 = s

(123)

Это совсем не очевидно, но позже мы покажем, что векторное (122) и спинорное (123) представления одного и того же абстрактного вращающегося объекта идентичны, точнее изоморфны. Пока что воспримем эквивалентность (122) и (123) как нечто заданное.

При учете спина в формализме НРФГ соответствующие уравнения, естественно, остаются прежними, а вот порядок всех матриц - гамильтониана [Н], контактных [¿], неравновесной функции Грина [Оп], спектральной функции [А] [19] удваивается за счет того, что электроны, находящиеся в одном и том же квантовом координатном состоянии, но с разными спинами, теперь описываются разными базисными функциями. Если раньше, когда не учитывалось различие между спинами двух электронов, находящихся в одном и том же квантовом состоянии, число базисных функций было равно Ы, то теперь число базисных функций равно 2Ы. В квантовой химии этот подход известен как неограниченный (по спину) метод

y

Хартри- Фока (Unrestricted Hartee - Fock) [48-49] или иначе метод разных орбиталей для разных спинов (Different Orbitals for Different Spins) [50] или расщепленных орбиталей [51].

В простейшем варианте описания канала проводимости без потери «физики», к чему мы всегда стремимся, в хюккелевском приближении, более известном в физической литературе как методе сильной связи в ортогональном базисе с учетом взаимодействия лишь соседних атомов, канал проводимости считается не чувствительным к спину, другими словами, одинаково взаимодействует с электронами, находящимися в разных спиновых состояниях. Гамильтониан [Н] проводника в хюккелевском приближении теперь запишется точно так же, как и в [19], только кулоновский е и резонансный t интегралы пишутся теперь в виде матриц 2 х 2 [а] и [в] (рис. 37), а именно:

М = 41 ]. [А ] = |А ] = t [ i ],

(124)

iНе можете найти то, что вам нужно? Попробуйте сервис подбора литературы.

где [I] - единичная матрица формы 2 х 2. Фактически налицо два расцепленных гамильтониана, что не несет какой-либо новой «физики».

Рис. 37. Однородная 2D решетка с параметром решетки а, в которой каждый атом описывается кулоновским [а] и резонансными [в интегралами в виде матриц 2 х 2

Аналогично записываются матрицы собственной энергии [¿] контактов, которые в этом приближении одинаково реагируют на разные спины, перемножая их обычные выражения [19] на единичную матрицу [12]. В этом приближении результат ожидается тривиальным в том смысле, что мы как бы учитываем лишь спины одного сорта, и лишь в конце вычислений все результаты просто удваиваем.

Все спиновые эффекты, рассмотренные, например, в [21], возникают либо как результат учета нетривиальных контактов, чувствительных к спину и описываемых соответствующими контактными матрицами [¿], либо канал проводимости чувствителен к спину и описывается соответствующим гамильтонианом [Н], либо имеют место оба обстоятельства.

Сейчас же пока рассмотрим несколько модельных транспортных задач методом НРФГ с уче-

том спина и начнем с простой одноуровневой модели спинового вентиля. С вычислительной точки зрения вопрос лишь в том, как записать гамильтониан и контактные матрицы, а далее ход вычислений стандартный [19].

11. Одноуровневый спиновый вентиль

Как известно [21], спиновый вентиль обнаруживает различную проводимость GP или GAP в зависимости от того, намагниченность обоих контактов параллельна Р или антипараллельна АР. Магнитосо-противление

MR = (Gp - Gap ) / Ga

(125)

выражается через поляризацию Р магнитных контактов

MR =

P

1 - P

(126)

которая в свою очередь определяется граничними сопротивлениями вентиля

P =

R - r R + r

(127)

при условии равенства нулю сопротивления канала проводимости.

В режиме магнитного туннельного перехода (МТП) в выражении для магнитосопротивления диэлектрического проводника появляется дополнительная двойка [21]:

MR =

2P 1 - P2

(128)

появление которой можно понять из физики туннельных проводников, согласно которой результирующее сопротивление двух последовательно соединенных проводников пропорционально их произведению, а не сумме.

Мы сейчас получим этот же результат, включая множитель двойку, применив метод НРФГ к одноуровневому вентилю в предположении, что равновесный электрохимический потенциал ц0 лежит на много кТ ниже энергии е диэлектрического проводника (рис. 38).

Рис. 38. Модель равновесного диэлектрического одноуровневого проводника

Согласно квантовой модели одноуровневого однородного Ш резистора без учета спина [19] гамильтониан и контактные матрицы есть просто числа (рис. 39).

Рис. 39. Однородный одноуровневый Ш проводник без учета спина

Собственные энергии контактов

=Й,2-

(129)

пропорциональны скоростям, с которыми электроны покидают проводник в направлении контактов [19].

При учете спина число базисных функций удваивается, вместо чисел появляются матрицы 2-го порядка: гамильтониан, однако, есть просто диагональная матрица, а вот контактные матрицы различные для спинов «вверх» и и спинов «вниз» ё (рис. 40).

Рис. 40. Однородный одноуровневый Ш проводник с учетом спина (спиновый вентиль)

Для спинового вентиля сразу получаем запаздывающую функцию Грина [19]

G ]=

E-е + г~ ^и +Уш )

Е-£+ i (yld +У2 d )

(130)

и далее коэффициент прохождения

т = тг ок г2 оа ] =

_ У\ыУ 2и , у\ау2а

(E-е)2 2 (E-е)2 + (У^У.)

•(131)

Нужно помнить, что сейчас мы рассматриваем «не чувствительный к спину» канал проводимости.

Для параллельной конфигурации намагниченности контактов Р оба контакта можно считать одинаковыми, так что (а>в)

У и = У 2 u = a У и = У2 d =

(132)

тогда как для антипараллельной конфигурации АР второй контакт опрокидывает спин по сравнению с параллельной конфигурацией Р, а именно:

Ухи = У 2 d = ^ Уld = У 2 u = ß.

(133)

Подставляя (132) в (128), получаем коэффициент Р-прохождения

Tp =-

ß2

(E-е) +а2 (E-е) + ß2 а (133) дает коеффициент ^^-прохождения

2aß

(134)

t =

TAP

(E-е)2 )2

Измеряемая проводимость определяется усредненным значением коэффициента прохождения в области энергий нескольких кТ вокруг значения ц0. Мы сейчас рассматриваем диэлектрический проводник (рис. 38). Предполагая

получаем

е — » кТ, а, ß,

gp~tp{e = m 0) =

а2 /?2

(ßo -е) + a (ßo -е) +ß2

а2 +ß2

(135)

(136)

(ßo -е)2

GAP ~Т4Р(Е = НО)'

2aß

(ßo -е)

а для магнитосопротивления получаем искомую формулу (128)

g а + в2 2Р2

MR = GL-1 = а +ß -1 = . 2P

G.

с поляризацией

p =

2aß

a-ß a+ ß

1 - P2

(137)

(138)

Точно так же можно получить формулу для маг-нитосопротивления (126) металлического немагнитного проводника, если предположить, что химпотенциал ц0 лежит вблизи уровня с энергией е, а кТ>>а, р.

и

12. Вращение магнитных контактов

В спиновом вентиле в режиме АР второй контакт опрокидывает спины по сравнению с вентилем в режиме Р, так что

Г =

* c - s a 0" ** c s

* s c _ 0 ß\ -s c

Г, =

"a 0" ~ß 0"

, Г =

_ 0 ß_ > ¿ 0 a

(139)

* c -s ac as

s c* -ßs ßc _

(145)

iНе можете найти то, что вам нужно? Попробуйте сервис подбора литературы.

Как записать матрицу Г для контакта, ориентированного в произвольном направлении единичного вектора п через его проекции (122) вдоль осей x, у, z ? Окончательная формула имеет вид

T_a+ß| a-ß

n + ш„

П - in,,

- n

(140)

acc + ßss* (a-ß cs* (a-ß) sc* ass* + ßcc

Используя определение компонент спинора c и 5 (123) и тригонометрических равенств

2 cos2 — = 1 + cos — 2 sin2 — = 1 - cos t 2 2

где I - единичная матрица 2-го порядка. Формула сразу не очевидна, но мы скоро ее получим. А пока что убедитесь, что эта формула содержит част -ные случаи параллельной и антипараллельной ориентации намагниченностей контактов, соот- р. ветственно:

и 2 sin—cos2 — = sin$, 2 2

перепишем (145) в виде

nz = +1, nx = ny = 0 : Г =

nz = -1, nx = ny = 0 : Г =

a 0

.0 ß.

ß 0 0a

(141)

Убедиться в справедливости общей формулы (140) можно путем следующего наблюдения. Матрица

Г =

ос 0 0 ß.

(142)

описывает магнитный контакт, ориентированный вдоль произвольно направленного единичного вектора п, если в качестве оси отсчета выбрать направления + п и - п вместо стандартных декартовых полуосей + Т и - Т . Вернуться назад к стандартному базису ±г можно путем унитарного преобразования Г=К Г Г матрицы (142):

n -n

z - z

z * c -s n a 0" n c* s*

- z * s c -n 0 ß_ -n -s c

(143)

v

V+

где колонки матрицы унитарного преобразования [V есть компоненты спинора с и 5 (123) вдоль направлений ± п . Первая колонка матрицы V соответствует направлению + п, а вторая колонка соответствует направлению - п с компонентами спинора -5* и с*, которые получаются из (123) путем преобразования углов

в^ж-в и ф^ж + ф

(144)

и удаления из обеих компонент общего фазового множителя.

Перемножение трех матриц в (143) дает

(a + ß + (a- ß cos— (a-ß) sin ве-ф (a-ß) sin ве+ф (a + ß)-(a-ß) cos —

(146)

(147)

что приводит к искомой формуле (140), если воспользоваться выражениями (122) для компонент единичного вектора п вдоль осей х, у, z. Если вектор поляризации

a + ß

(148)

определить его величиной (138) и единичным вектором п, то уравнение (140) можно переписать в виде

Г =

a + ß

2

[ I ] +

P + iP

P - iPy

- P

(149)

или же еще иначе

[г]

(a + ß)/2

+P

1 0" "0 1"

+P

0 1 x 1 0

" 0 -i 1 0"

+ P

+i 0 0 -1

(150)

через широко используемые в квантовой теории спиновые матрицы Паули

. (151)

В конечном итоге уравнение (149) можно переписать компактно в виде

"0 1" " 0 -i "1 0 "

1 0 ' y +i 0 > z 0 -1

Г =

a + ßi

(И+кк+Ы^+КИ)

(152)

Подобным же образом можно компактно переписать контактные матрицы собственной энергии. Пусть, например, в базисе ± п

+

=

2

Z = — 2

a 0 ß.

(153)

тогда в базисе ± z имеем:

1 =

(154)

13. Спиновые гамильтонианы

Перейдем к обсуждению спин-зависимых гамильтонианов и рассмотрим учет классического спин-орбитального зеемановского расщепления и гамильтониан Рашбы [52, 53].

13. 1. Гамильтониан с зеемановским расщеплением. Пусть направление магнитного поля

В определяется единичным вектором п. Тогда гамильтониан в базисе ± п имеет вид

+B 0 0 -B

(155)

где ¡1а - магнитный момент электрона. Уровень с положительным спином смещается вверх на величину цаВ, а уровень с отрицательным спином

смещается вниз на эту же величину (зеемановское расщепление).

В базисе ± г имеем:

Нв = ¡лер В .

(156)

Добавить еще нужно спин-независимую часть гамильтониана. Для случая параболической дисперсии имеем

Н=^(к2х+к2у)[1] + ъ*-В, (157)

а для однородной 2D решетки в хюккелевском приближении с резонансным и кулоновским интегралами [19]

t = — h / 2та

s = Е - 4t

(158)

(159)

гамильтониан имеет вид:

Н = {е + 1Лса&кха + 11(Х)$куа} [/] +

iНе можете найти то, что вам нужно? Попробуйте сервис подбора литературы.

(160)

где а - параметр решетки. Для такой решетки (рис. 37) кулоновская и резонансная матрицы имеют вид:

а = е[/] + jUda-B ßx = t [/], ßy= t [I ].

(161)

По сравнению со спин-независимым гамильтонианом (124) изменяется только кулоновская матрица.

13. 2. Гамильтониан Рашбы

Более полный учет спин-орбитального взаимодействия дает гамильтониан

Hr = Ф ■ (<? х к) = r¡ (ахку -аукх) :

(162)

где п - параметр Рашбы [52-54]. Этот гамильтониан имеет релятивистское происхождение, обусловленные им эффекты широко наблюдаются экспериментально и надежно интерпретированы [54]. Нас же сейчас интересует только одно - как учесть взаимодействие Рашбы для нашей однородной 2D решетки в хюккелевском приближении. С этой целью аппроксимируем (162) синусами

HR = —(ох sin ka — о sin kxa) (163)

av y y '

и запишем их удобства ради через экспоненты

Н =

— I + ik a - ik a

(

2ia v

2ia Л

+ika -ika |

e k - e k I

(164)

сомножители перед которыми

— —

В = —о , В =---о

Px 2a y' Px 2a y

i— i—

P, =---ox, Pl = ox

y 2a x y 2a x

(165)

обеспечивают правильное дисперсионное соотношение (157).

Учтем спин-независимую часть гамильтониана 2D решетки (4), а также постоянное магнитное поле (41) через зеемановское расщепление. Окончательно кулоновская и резонансная матрицы однородной 2D решетки с учетом зеемановского спин-орбитального взаимодействия и взаимодействия Рашбы имеют вид:

а = е1 + ¿ие1а-В,

Р% = и + — а , Д+= и - — <зу, х 2а у х 2а у

ß = ti - — ах, ß+ = ti + — ах

y 2a x y 2a x

(166)

Обратим внимание на то, что взаимодействие Рашбы учитывается через параметр Рашбы и матрицы Паули в резонансных матрицах 2D решетки, а зеемановское взаимодействие - в кулоновских матрицах.

14. Спиноры и векторы

Мы обычно визуально представляем себе спин как вращающийся объект, ориентированный в определенном направлении. Он описывается двумя комплексными компонентами спинора (123), а вектор в этом же направлении задается тремя действительными проекциями (122) на оси декартовой системы координат. Чтобы почувствовать связь между двумя представлениями одного и того же направленного объекта - векторным и спинорным полезно рассмот-

реть прецессию спина в магнитном поле в обоих представлениях.

Рассмотрим одноуровневый резистор с е = 0 в магнитном поле, направленном вдоль оси z. Двух-компонентное уравнение Шредингера

d Wu dt [w

ИЛ

iti

1 0

0 -1

(167)

где справа стоит спиновая матрица аг, фактически есть два дифференциальных уравнения с очевидными решениями

¥u{t )=М 0) е -(t)=¥* (0) е+ где частота прецессии

ю = -

(168)

(169)

Пусть спин электрона в начальный момент времени t = 0 направлен вдоль единичного вектора п под углами в и ф относительно выбранной системы отсчета (рис. 41).

Рис. 41. Выбор системы координат для описания прецессии по углу ф спина, направленного вдоль единичного вектора п

В этот начальный момент времени его волновые функции

W (0) = cos — е-ф/2 W (0) = sin—е+ф/2

а в последующие моменты времени

(170)

(Л „„„ в -фФ/2 -imt/2 Wu (t)= co^ e Ф e

+¡ф/2 + ict/2

Wd (t) = sin—е+ф/2е

(171)

что соответствует вращению спина вокруг оси z под фиксированным к ней углом в, тогда как угол ф растет линейно со временем:

ф(/) = ф(0) + ю/ .

(172)

Согласно (122), для проекций единичного вектора имеем:

пх = sin ecos ф(^) , и = sin в sin ф(/) ,

И = cose, (173)

откуда с использованием (172) получаем

dn dn

—- = -ю n , —— = +ю n dt y dt

(174)

или в матричном виде для всех трех компонент единичного вектора

d_

dt

n* " 0 -1 0" nx

ny > = ю +1 0 0 ■ ny

Л 0 0 0 Л ,

(175)

где квадратная матрица, обозначаемая обычно Я2, есть матрица вращения единичного вектора вокруг оси z.

Сравните полученное уравнение (125) с уравнением Шредингера (167) в спинорном представлении, переписанном через частоту прецессии (169):

d_

dt

1 0 0 -1

(176)

где фигурирует матрица Паули аг.

Пусть мы хотим описать прецессию спина электрона в магнитном поле, направленном вдоль оси х. Для этого в уравнении (175) достаточно лишь выполнить циклическую перестановку переменных: х—у, y—z, z—>х и мы получим уравнение с матрицей вращения Ях:

d_

dt

nx " 0 0 0 " nx

ny • = ю 0 0 -1 ■ ny

Л 0 +1 0 _ л,

(177)

если в уравнении (175) -1 стояла на пересечении строки х и колонки у, то в (177) она окажется на пересечении строки у и колонки z.

Сразу не очевидно как именно с той же целью модифицировать уравнение (176). Правильный ответ таков:

d_ Wu

dt [w

0 1 1 0

(178)

iНе можете найти то, что вам нужно? Попробуйте сервис подбора литературы.

где фигурирует матрица Паули ах.

Уравнения (175) и (176), как и уравнения (177) и (178), описывают один и тот же физический процесс, а именно, вращение спина электрона в магнитном поле, направленном вдоль оси z, соответственно вдоль оси х, в первом случае - через три действительных компоненты, а во втором - через две комплексных. Циклическая перестановка в противоположном направлении даст нам матрицу вращений Яу, а соответствующая ей спиновая матрица оу приведена в сводке (151).

Что делает матрицы вращения и спиновые матрицы изоморфными, позволяя им описывать один

и тот же физический процесс? Ответ состоит в том, что у этих матриц одинаковые коммутационные соотношения.

Легко убедиться, что матрицы вращения

"0 0 0

Rx = 0 0 -1

0 +1 0

" 0 0 +1

Ry = 0 0 0

-1 0 0

" 0 -1 0"

Rz = +1 0 0

0 0 0

(179)

подчиняются следующим коммутационным соотношениям:

[*х , ку > кхку - кукх = ъ ,

[ ку , я; яуяг - я;яу = ях ,

[я;, ях > я;ях - яхя; = яу, (180)

где слева стоят стандартные обозначения соответствующих коммутаторов.

Спиновые матрицы Паули подчиняются таким же коммутационным соотношениям, если Я формально заменить на а/2г.

[ах ,ау ~]= ахау - ауах = 2'а; ,

[Яу , а; ] = ауЯ; -а2&у = ^ ,

К ,ах ] = а;ах -аха; = 2'Яу . (181)

Заметим, что в стандартных учебниках квантовой механики спиновые матрицы Паули обычно вводятся постулативно.

Какие должны быть компоненты у спинора, направленного вдоль оси I? Поскольку спинор не изменяется при вращении вокруг оси г, то его компонентами должны быть собственные векторы спиновой матрицы а2 в сводке (151):

11 (0

> или | 01 |1

(182)

представляющие действительно спин «вверх» или спин «вниз» вдоль оси г.

Если же мы хотим узнать компоненты спинора, направленного вдоль оси х, то нужно найти собственные векторы спиновой матрицы ах , каковыми являются векторы

ш

или

Ш.

(183)

представляющие направление спина «вверх» вдоль положительной полуоси или спина «вниз» вдоль отрицательной полуоси оси х.

В общем случае, если мы хотим найти компоненты спинора, направленного вдоль произвольно

ориентированного единичного вектора n (122), нужно искать собственные векторы матрицы

ст• w = ах sin0cos ф +

+ст sin ö sin ^ + az cosö =

cos^

sin вг+'ф

обозначенные ранее как

sin öe^ - cos в

(184)

первый из которых соответствует ориентации спина вдоль положительного направления вектора п и в явном виде выписан в (123).

15. Прецессия спина

Мы уже знаем, как составить гамильтониан Н и контактные матрицы £, в том числе и с учетом нетривиальных спиновых эффектов, и готовы просчитать численные модели вычисляя электронную плотность через неравновесную функцию Грина О", плотность состояний через спектральную функцию А и ток [19].

Рис. 42. Схемы измерения и расчета методом НРФГ спинового потенциала в рамках четырехтерминальной модели однородного Ш проводника с двумя магнитными зондами, описываемыми контактными матрицами Е] и Е2

Сначала рассчитаем потенциал на 2-ом зонде, описываемом контактной матрицей Е2, однородного Ш проводника (рис. 42) методом НРФГ с простым гамильтонианом без учета спиновых эффектов в зависимости от угла наклона в магнитного зонда 2 по отношению к линейному проводнику (плавающий контакт). Результаты расчета для различных значений поляризации (127) Р2 2-го зонда относительно 1-го представлены на рис. 43.

Полностью подтверждается расчетом методом НРФГ формула (121): измеряемый потенциал осциллирует в зависимости от угла наклона одного магнитного контакта относительно другого. При нулевой поляризации контактов измеряемый потенциал остается постоянным.

Как уже упоминалось вначале, нет сведений, что подобные эксперименты с плавающим магнит-

и

ным контактом действительно проводились. Проводились, однако, эксперименты с вращением спина электронов в канале проводимости при фиксированной геометрии взаимного расположения магнитных контактов.

Рис. 43. Схема измерения с плавающим зондом 2 и изменение потенциала 2-го зонда относительно 1 -го в зависимости от угла наклона, нормированного на 180°, при разных значениях взаимной поляризации зондов [13]

Один из широко распространенных методов заключается в приложении внешнего магнитного поля, что приводит к прецессии спина относительно направления магнитного поля с циклической частотой ю (169). Это означает, что спиновый потенциал в точке, где находится пробный зонд 2, повернется на угол тп, где п есть время прохождения электроном пути от инжектирующего контакта до пробного зонда 2. Записав п=Ь/у, из уравнений (121) и (169) имеем:

2и.L

¡лг — /л + Р2/лs cos —В_.

яг

(185)

Можно ожидать появления осциллирующего потенциала на зонде 2, зависящего от величины приложенного магнитного поля. Однако, обычно наблюдают не баллистический транспорт, и имеет место большой разброс в значениях времен п, в результате чего потенциал, усредненный по всем временам п зануляется. Типичные эксперименты вместо осциллирующего сигнала, зависящего от магнитного поля, показывают просто уменьшение потенциала от значения Р2ц5 вплоть до нуля (эффект Хэнли [55, 56]). И тем не менее, осцилляции в эффекте Хэнли наблюдать удалось за счет уменьшения разброса времен п настолько, что разброс оказался намного меньше среднего значения времени п [57].

Другим подходом к вращению спина электрона может служить эффект Рашбы в проводниках с сильным спин-орбитальным взаимодействием. Сейчас хорошо установлено на примере многих полупроводников, что электрическое поле вдоль оси z

(рис. 42) ведет к появлению эффективного магнитного поля, зависящего от импульса электрона. В этом можно убедиться, сравнивая гамильтониан с учетом магнитного поля (156) с гамильтонианом Рашбы (162), который, удобства ради, перепишем в виде

HR =r¡z-(o'xk^ = r¡5 ■ (ä' x z),

(186)

откуда для эффективного магнитного поля, обязанного взаимодействию Рашбы, получаем

HeAff =r¡zxk.

(187)

так что согласно (185) можно ожидать осциллирующий потенциал вида

Hl =И + P2 Hs cos

2r¡kL hv

(188)

с периодом

2kL

-Atj = 2 л

hv

или иначе

2natn Ап=-sin ka.

kL

(189)

Этот вывод находится в неплохом согласии с численными результатами, полученными методом НРФГ для энергии, соответствующей ка = ж/3 при расстоянии между инжектором и детектором Ь = 40а (рис. 44).

Рис. 44. Схема измерения с плавающим зондом 2 в проводниках с большим значением коэффициента Рашбы п и осцилляционная зависимость потенциала зонда 2 когда намагниченности обоих зондов направлены вдоль проводника (ось х), если же намагниченности направлены перпендикулярно к проводнику (ось z), то осцилляции исчезают [13]

В этой модели электроны, движущиеся вдоль оси х в ее положительном направлении, испытывают влияние эффективного магнитного поля вдоль оси у. Поскольку инжектируемые электроны имеют спино-

выи потенциал /л , направленный вдоль намагниченности зондов 1 и 2 (ось х), спиновый потенциал должен вращаться. Осцилляции потенциала исчезают, если намагниченности обоих зондов направлены вдоль оси у. Этот эффект подтверждается экспериментально [58, 59].

16. Квантовый спиновый эффект Холла

Транспорт поляризованных спинов в немагнитных материалах, таких как медь, изучен достаточно полно и физические основы в целом ясны. Этого нельзя сказать о диффузии спинов в материалах с сильным спин-орбитальным взаимодействием.

В последнее десятилетие пристальное внимание уделялось квантовому спиновому эффекту Холла (КСХ). В обычном классическом эффекте Холла электроны, движущиеся от истока к стоку вдоль оси х, под действием магнитного поля, направленного перпендикулярно поверхности 2D проводника, вдоль оси г, закручиваются таким образом, что создается поперечная разность потенциалов вдоль оси у (рис. 45).

Рис. 45. Выбор системы координат при обсуждении эффекта КСХ

Эффект КСХ можно понять в рамках модели, согласно которой эффективное магнитное поле закручивает спины, поляризованные вдоль полуосей +г и -г, в противоположных направлениях, тогда как в классическом эффекте Холла оба сорта спинов закручиваются в одном и том же направлении.

Поскольку электроны в проводниках с сильным спин-орбитальным взаимодействием испытывают действие эффективного магнитного поля, естественно задать вопрос, можно ли наблюдать холлов-ский эффект в отсутствии внешнего магнитного поля, а лишь под влиянием внутреннего эффективного магнитного поля.

Ответ оказывается положительным, но подобное явление с закручиванием спинов, поляризованных вдоль +г и -г, в противоположных направлениях представляется весьма деликатным явлением, по-скольску заряды не накапливаются в поперечном направлении и холловское напряжение зануляется. Однако, спиновый потенциал может быть измерен магнитнымы контактами.

В первых экспериментах накопление спинов противоположной поляризации в поперечном направлении (КСХ) обнаружили в массивных полупроводниках оптическими зондами [60], а недавно КСХ этого типа измерили магнитными зондами в нанопроводниках [59].

Транспортная модель на основе метода НРФГ [19] адекватно описывает КСХ, о чем свидетельствуют результаты расчета однородного 2D проводника (рис. 44) с коэффициентом Рашбы п= =110-11 эВм с энергией Е=0.05t0 и переменной ширине Ж (рис. 46). Для вычленения г-компоненты спиновой плотности из гриновской функции О" использовалось уравнение (185).

В последние несколько лет значительно возрос интерес к транспорту спинов в материалах с сильным спин-орбитальным взаимодействием, особенно в топологических диэлектриках [21], которые обнаруживают совсем удивительные проявления КСХ [60].

Рис. 46. Квантовый спиновый эффект Холла. Численные результаты, полученные методом НРФГ, показывают накапливание +г-спинов и -г-спинов £(г) (в произвольных единицах) на противоположных сторонах 2D проводника, при этом изменение направления тока вдоль проводника на противоположное приводит к обращению накапливаемых спинов [13]

17. Метод НРФГ и диффузионный транспорт

В заключение обсудим связь метода НРФГ с диффузионным подходом для баллистического транспорта, развитым в [21]. Мы уже знаем, что численные результаты, полученные выше методом НРФГ, качественно верно описываются уравнением (121), сформулированным в [21] из эвристических соображений. Однако, причина этого согласия не очевидна, тем более что в методе НРФГ для моделирования транспорта спинов первично используются двухкомпонентные комплексные спиноры, а спиновый потенциал ]л* описывается трехкомпонентным действительным вектором.

Выше уже подчеркивалась связь между единичным вектором ", вдоль которого направлен спин, и спиновыми волновыми функциями фир и Чтобы установить связь между уравнением (121) и методом НРФГ для транспорта спинов, нужно рассматривать такие величины как гриновская функция О" ~ цпу+, а не просто волновая функция ф, поскольку метод НРФГ исходно формулируется через о" . Не говоря уже о том, что именно гриновская функция о", а не волновая функция, является наблюдаемой величиной, пригодной для установления связи с экспериментальными результатами.

17. 1. Электронная плотность в матричном представлении

Мы уже не раз использовали электронную плотность в матричной записи [ О" ], диагональные элементы которой дают нам число электронов в данной точке проводника [19]. С учетом спина матрица [ О" ] в данной точке есть матрица (2x2) и она дает

iНе можете найти то, что вам нужно? Попробуйте сервис подбора литературы.

нам число электронов или суммарный спин . Убедиться в этом можно рассматривая электрон со спином, направленным вдоль произвольного направления " (рис. 41) и описываемым волновой функцией в спинорном представлении (170), так что соответствующая матрица электронной плотности [ О" ] формы (2 х 2) дается выражением

WW+ =

*• ¡=

cc cs

sc ss

или иначе, используя (122) и (146),

WW =-

1

1 + n2 п + in,.

П - in у 1 - n.

= -[l + cr-n

(190)

]-(191)

Суммируя вклады в от всех N электро-

нов, для матрицы гриновской функции окончательно получим

N + S S - iSv

Z X у

S + iS N - S_

+ er-S). (192)

Вычислив матрицу [ О" ], полное число электронов и суммарный спин можно найти из

N = — Tr{Ga\ S=— Tr[aG"l 2n l j 2 ж L J

(193)

что следует из равенства нулю следа спиновых матриц Паули (151) и из следующих свойств спиновых матриц, вытекающих из (151) и (181), а именно:

<? = а2 = а2 = i,

(194)

полное уравнение (121) в векторной форме, опираясь на метод НРФГ (рис. 47).

Рис. 47. Моделирование детектирующего зонда в полуклассической модели и в методе НРФГ

Для тока имеем четвертое уравнение в формализме НРФГ (ур-е (16) в [19])

/~Г/-[Г][/«[Л]-[е»]], (195)

так что при нулевом токе на детектирующем зонде (рис. 47)

fiet

Tr [r][Gn ]

Tr[Г][A] •

(196)

Подставляя гриновскую функцию О" (192), спектральную функцию А выражая через спин-независимую плотность состояний Б

- = - [ I ],

2п 2

(197)

описывая связь детектирующего зонда с каналом проводимости согласно (152)

(198)

и учитывая нулевой след у всех матриц Паули, окон-чате ль но получим:

/det =Tr[l + ä-P]

NT ^ S —/+ст — D D

(199)

UzUx = -UxUz = Uy •

В итоге, вся информация, содержащаяся в эрмитовой матрице [ О" ], может быть представлена четырьмя действительными величинами в данной точке проводника - полным числом электронов N и тремя действительными компонентами вектора полного спина , которые можно извлечь из (193).

17. 2. Измерение спинового потенциала

Мы подошли к установлению связи между уравнением (121) и методом НРФГ для транспорта спинов. Скалярную версию уравнения (121) (ур-е (53) в [21]) мы получили в рамках полуклассической модели вычисления токов на детектирующем зонде через его граничные проводимости для спинов противоположной направленности (рис. 12 в [21]), полагая зондовый ток нулевым. Теперь мы получим

Учитывая тождество для двух произвольных векторов Р и В = §/Б

[ст • р] [ст • я] = (Р ■ в) [/] + /ст • [Р X я] , (200)

имеем

[l + a-P~][bI + a-B~] = ■■{b + P-B)[l] + ä-[P + B + iPxB

(201)

и окончательно, используя равенство нулю следа матриц Паули, вместо (199) имеем:

N

S

fä«=- + P — = f + P

D

D

А 2

(202)

где зарядовая заселенность /и спиновая определены таким образом, чтобы их максимально возмож-

UxUy = = UZ

а а = -u а = ia

у z z у x

ные значения не превышали единицы. Чтобы перейти от заселенностей к химпотенциалам и получить искомое уравнение (121), воспользуемся линейной зависимостью между ними в режиме линейного отклика ((21) в [14]).

17. 3. Четырехкомпонентный транспорт

Подчеркнем еще раз, что квантовый подход естественным образом ведет к комплексной матрице [ о" ] формы (2 х 2) в каждой точке проводника, и вместе с тем гриновскую матрицу всегда можно переписать через четыре действительных и физически понятных числа таких как (V. ) или (и. ¡й*).

Мы уже видели в [21] как много различных явлений спинового транспорта можно описать с помощью уравнения Вале-Фера (ур-е (23) в [21]) вместе с моделью спин-зависимых пограничных сопротивлений. Однако, этот подход [21] был ограничен применением к задачам транспорта спинов только в одном направлении (ось X). Сейчас же, имея полную версию метода НРФГ, можно рассматривать транспорт спинов, ориентированных в любом произвольном направлении, хотя подобные расчеты и требуют больших вычислительных ресурсов.

Транслируя матрицу | (I" | в четырехкомпонентный формат (Ы, 51) спин-зависимые граничные проводимости [21] могут быть замещены на матрицу проводимости О формы (4 х 4), которая свяжет четыре компоненты потенциалов с четырьмя компонентами тока:

(203)

Аналогичным образом, двухкомпонентное уравнение Вале-Фера для (м, м) может быть конвертировано в четырехкомпонентное уравнение диффузии ДЛЯ (//, fis ).

Как уже упоминалось выше, учет различия между двумя состояниями спина удваивает порядок всех матриц в методе НРФГ. Например, если в 1D проводнике учитываются три атома, то блочная матрица [ gn ] будет 6-го порядка:

1 1 2 2 3 3

up dn up dn up dn

I А/

1' sx ' g ' AUsx

4 x 4 AUsy

isz AUsz .

\"P [ВД] 1

[n2,s2]

(204)

3

[>зД]

в которой диагональные блоки (2 х 2) соответствуют четырехкомпонентному формату описания транспорта.

Чего не хватает в гриновской матрице (204), так это недиагональных элементов, связывающих соседние и более удаленные друг от друга атомы проводника. Эти элементы гриновской матрицы порождают эффекты квантовой интерференции. Для некоторых транспортных задач это не существенно, поскольку эти эффекты часто нивелируются процессами дефазировки [19].

18. Выводы

В рамках концепции «снизу - вверх» современной наноэлектроники рассмотрены классический и квантовые эффекты Холла, методы измерения электрохимических потенциалов, формулы Ландауэ-ра и Бюттекера, измерение холловского потенциала, учет магнитного поля в методе НРФГ, уровни Ландау и краевые состояния в графене, спиновый транспорт в формализме НРФГ в спинорном представлении, в частности, подробно обсуждены спиновый вентиль, вращение магнитных контактов, прецессия спина и вращение спинов, роль спиновых гамильтонианов Зеемана и Рашбы, квантовый спиновый эффект Холла, вычисление спинового потенциала, а также четы-рехкомпонентный формат описания спинового транспорта.

Благодарности

Я благодарен проф. С. Датта (Supriyo Datta) за возможность прослушать его курсы лекций «Fundamentals of Nanoelectronics, Part I: Basic Concepts» и «Fundamentals of Nanoelectronics, Part II: Quantum Models», прочитанных он-лайн в январе - апреле 2012 года и в марте - мае, октябре - декабре 2015 года в рамках инициативы Purdue University/nanoHUB-U [2012 [61]; 2015 [62]] и частично положенных в основу настоящего обзора.

Я также благодарен моей жене Н. Е. Кругляк за подготовку рисунков к публикации и помощь в оформлении рукописи.

Литература

1. Hall, E. H. On a New Action of the Magnet on Electric Currents [Text] / E. H. Hall // American Journal of Mathematics. - 1879. - Vol. 2, Issue 3. - P. 287-292. doi: 10.2307/ 2369245

2. Klitzing, K. V. New Method for High-Accuracy Determination of the Fine-Structure Constant Based on Quantized Hall Resistance [Text] / K. V. Klitzing, G. Dorda, M. Pepper // Physical Review Letters. - 1980. - Vol. 45, Issue 6. - P. 494497. doi: 10.1103/physrevlett.45.494

3. фон Клитцинг, К. Квантовый эффект Холла: Нобелевские лекции по физике [Текст] / К. фон Клитцинг // УФН. - 1985. - Т. 150, №. 1. - С. 107-126.

4. Reedtz, G. M. An automated potentiometric system for precision measurement of the quantized hall resistance [Text] / G. M. Reedtz, M. E. Cage // Journal of Research of the National Bureau of Standards. - 1987. - Vol. 92, Issue 5. -P. 303-310. doi: 10.6028/jres.092.030

5. Tsui, D. S. Two-dimensional magnetotransport in the extreme quantum limit [Text] / D. C. Tsui, H. L. Stormer, A. C. Gossard // Physical Review Letters. - 1982. - Vol. 48, Issue 22. - P. 1559-1962. doi: 10.1103/physrevlett.48.1559

2

up

iНе можете найти то, что вам нужно? Попробуйте сервис подбора литературы.

2

dn

3

up

6. Степановский, Ю. П. Дробный квантовый эффект Холла [Текст] / Ю. П. Степановский // Электромагнитные явления. - 1998. - Т. 1, № 3. - С. 427-442.

7. Nagaosa, N. Anomalous Hall effect [Text] / N. Nagaosa, J. Sinova, S. Onoda, A. H. MacDonald, N. P. Ong // Reviews of Modern Physics. - 2010. - Vol. 82, Issue 2. -1539-1592. doi: 10.1103/revmodphys.82.1539

8. Dyakonov, M. I. Possibility of orientating electron spins with current [Text] / M. I. Dyakonov, V. I. Perel // Sov. Phys. JETP Lett. - 1971. - Vol. 13. - P. 467.

9. Dyakonov, M. I. Current-induced spin orientation of electrons in semiconductors [Text] / M. I. Dyakonov, V. I. Perel // Physics Letters A. - 1971. - Vol. 35, Issue 6. -P. 459-460. doi: 10.1016/0375-9601(71)90196-4

10. Kane, C. L. Quantum Spin Hall Effect in Graphene [Text] / C. L. Kane, E. J. Mele // Physical Review Letters. -2005. - Vol. 95, Issue 22. doi: 10.1103/physrevlett.95.226801

11. Srinivasan, S. All-Spin Logic Device With Inbuilt Nonreciprocity [Text] / S. Srinivasan, A. Sarkar, B. Behin-Aein, S. Datta // IEEE Transactions on Magnetics. - 2011. - Vol. 47, Issue 10. - P. 4026-4032. doi: 10.1109/tmag.2011.2159106

12. Kane, C. L. Topological Insulators [Text] / C. Kane, J. Moore // Physics World. - 2011. - Vol. 24, Issue 02. - P. 3236. doi: 10.1088/2058-7058/24/02/36

13. Datta, S. Lessons from Nanoelectronics: A New Perspective on Transport [Text] / S. Datta. - Hackensack, New Jersey: World Scientific Publishing Company, 2012. - 492 p. doi: 10.1142/8029

14. Кругляк, Ю. А. Наноэлектроника «снизу -вверх»: Возникновение тока, обобщенный закон Ома, упругий резистор, моды проводимости, термоэлектричество [Текст] / Ю. А. Кругляк // ScienceRise. - 2015. - T. 7, № 2 (12). - С. 76-100. doi: 10.15587/2313-8416.2015.45700

15. Кругляк Ю. О. Уроки наноелектрошки: виник-нення струму, формулювання закону Ома i моди провщносп в концепцй «знизу - вгору» [Текст] / Ю. О. Кругляк, Н. Ю. Кругляк, М. В. С^жа // Сенсор. електр. мжросист. техн. - 2012. - Т. 9, № 4. - С. 5-29. - Режим доступу: http://dspace.onu.edu.ua: 8080/handle/ 123456789/3643

16. Ашкрофт, Н. Физика твердого тела. Т. 1-2 [Текст] / Н. Ашкрофт, Н. Мермин. - М: Мир, 1979. - 486 c.

17. Rashba, E. I. Spin Currents in Thermodynamic Equilibrium: The Challenge of Discerning Transport Currents [Text] / E. I. Rashba // Physical Review B. - 2003. - Vol. 68, Issue 24. doi: 10.1103/physrevb.68.241315

18. Buttiker, M. Symmetry of Electrical Conduction [Text] / M. Buttiker // IBM Journal of Research and Development. - 1988. - Vol. 32, Issue 3. - P. 317-334. doi: 10.1147/ rd.323.0317

19. Кругляк, Ю. А. Наноэлектроника «снизу -вверх»: Метод неравновесных функций Грина, модельные транспортные задачи и квантовая интерференция [Текст] / Ю. А. Кругляк // ScienceRise. - 2015. - T. 9, № 2 (14). -С. 41-72. doi: 10.15587/2313-8416.2015.48827

20. Кругляк, Ю. А. Графен в транспортной модели Ландауэра - Датты - Лундстрома [Текст] / Ю. А. Кругляк // ScienceRise. - 2015. - Т. 2, № 2 (7). - С. 93-106. doi: 10.15587/2313-8416.2015.36443

21. Кругляк, Ю. А. Наноэлектроника «снизу -вверх»: Начала спинтроники и магнетроники [Текст] / Ю. А. Кругляк // ScienceRise. - 2015. - T. 8, № 2 (13). -С. 51-68. doi: 10.15587/2313-8416.2015.47792

22. Sears, F. W. Thermodynamics, Kinetic Theory, and Statistical Thermodynamics [Text] / F. W. Sears, G. L. Salinger. - Boston: Addison-Wesley, 1975.

23. Landauer, R. Spatial variation of currents and fields due to localized scatterers in metallic conduction [Text] / R. Landauer // IBM Journal of Research and Development. -1957. - Vol. 1, Issue 3. - P. 223-231. doi: 10.1147/rd.13.0223

24. Landauer, R. Electrical resistance of disordered one-dimensional lattices [Text] / R. Landauer // Philosophical Magazine. - 1970. - Vol. 21, Issue 172. - P. 863-867.

doi: 10.1080/14786437008238472

25. Laundauer, R. Spatial variation of currents and fields due to localized scatterers in metallic conduction [Text] / R. Laundauer // IBM Journal of Research and Development. - 1988. - Vol. 32, Issue 3. - P. 306-316. doi: 10.1147/rd.323.0306

26. Landauer, R. Spatial variation of currents and fields due to localized scatterers in metallic conduction (and comment) [Text] / R. Landauer // Journal of Mathematical Physics. -1996. - Vol. 37, Issue 10. - P. 5259. doi: 10.1063/1.531590

27. Buttiker, M. Four-terminal phase-coherent conductance [Text] / M. Buttiker // Physical Review Letters. -1986. - Vol. 57, Issue 14. - P. 1761-1764. doi: 10.1103/ physrevlett. 57.1761

28. Шарвин, Ю. В. Об одном возможном методе исследования поверхности Ферми [Текст] / Ю. В. Шарвин // ЖЭТФ. - 1965. - Т. 48, № 3. - С. 984-985.

29. Sharvin, Yu.V. Investigation of Focusing of Electron Beams in a Metal by a Longitudinal Magnetic Field [Text] / Yu. V. Sharvin, N. I. Bogatina // Zh. Eksp. Teor. Fiz. -1969. - Vol. 56, Issue 3. - P. 772-779.

30. Imry, Y. Directions in Condensed Matter Physics [Text] / Y. Imry; G. Grinstein, G. Mazenko (Eds.). - Singapore: World Scientific, 1986. - 101 p.

31. Imry, Y. Laundauer Rolf. Conductance viewed as transmission [Text] / Y. Imry, R. Landauer // Reviews of Modern Physics. - 1999. - Vol. 71, Issue 2. - P. S306 - S312. doi: 10.1103/revmodphys.71.s306

32. Лесовик, Г. Б. Описание квантового электронного транспорта с помощью матриц рассеяния [Текст] / Г. Б. Лесовик, И. А. Садовский // УФН. - 2011. - Т. 181, № 10. - С. 1041-1096. doi: 10.3367/ufnr.0181.201110b.1041

33. Stone, A. D. What is measured when you measure a resistance? - The Landauer formula revisited [Text] / A. D. Stone, A. Szafer // IBM Journal of Research and Development. - 1988. - Vol. 32, Issue 3. - P. 384-413. doi: 10.1147/ rd.323.0384

34. Mojarad, R. G. Atomistic non-equilibrium Green's function simulations of graphene nano-ribbons in the quantum hall regime [Text] / R. Golizadeh-Mojarad, A. N. M. Zainud-din, G. Klimeck, S. Datta // Journal of Computational Electronics. - 2008. - Vol. 7, Issue 3. - P. 407-410. doi: 10.1007/ s10825-008-0190-x

35. Haug, R. J. Edge-state transport and its experimental consequences in high magnetic fielgs [Text] / R. J. Haug // Semiconductor Science and Technology. - 1993. - Vol. 8, Issue 2. - P. 131-153. doi: 10.1088/0268-1242/8/2/001

36. Cage, M. E. Current Distributions in Quantum Hall Effect Devices [Text] / M. E. Cage // Journal of Research of the National Institute of Standards and Technology. - 1997. -Vol. 102, Issue 6. - P. 677-691. doi: 10.6028/jres.102.045

37. Мартинес-Дуарт, Дж. М. Нанотехнологии для микро- и оптоэлектроники [Текст] / Дж. М. Мартинес-Дуарт, Р. Дж. Мартин-Палма, Ф. Агулло-Руеда. - Москва: Техносфера, 2007. - 368 с.

38. Berger, C. Electronic Confinement and Coherence in Patterned Epitaxial Graphene [Text] / C. Berger, Z. Song, X. Li, X. Wu, N. Brown, C. Naud et. al. // Science. -2006. - Vol. 312, Issue 5777. - P. 1191-1196. doi: 10.1126/ science.1125925

39. Brey, L. Edge states and quantized Hall effect in Graphene [Text] / L. Brey, H. A. Fertig // Physical Review B. -2006. - Vol. 73, Issue 19. doi: 10.1103/physrevb.73.195408

40. Peres, N. M. Conductance quantization in mesoscop-ic graphene [Text] / N. M. R. Peres, A. H. Castro Neto, F. Guinea // Physical Review B. - 2006. - Vol. 73, Issue 19. doi: 10.1103/physrevb.73.195411

41. Abanin, D. A. Spin-filtered edge states and quantum Hall effect in Graphene [Text] / D. A. Abanin, P. A. Lee, L. S. Levitov // Physical Review Letters. - 2006. - Vol. 96, Issue 17. doi: 10.1103/physrevlett.96.176803

42. Kubo, R. Statistical-Mechanical Theory of Irreversible Processes. I. General Theory and Simple Applications to Magnetic and Conduction Problems [Text] / R. Kubo // Journal of the Physical Society of Japan. - 1957. - Vol. 12, Issue 6. -P. 570-586. doi: 10.1143/jpsj.12.570

43. Yourgrau, W. Treatise on Irreversible and Statistical Thermophysics [Text] / W. Yourgrau, A. van der Merwe, G. Raw. - Dover, New York, 1982.

44. Кругляк, Ю. А. Модель проводимости Ландауэ-ра - Датты - Лундстрома в микро- и наноэлектронике и транспортное уравнение Больцмана [Текст] / Ю. А. Кругляк // ScienceRise. - 2015. - T. 3, № 2 (8). - С. 108-116. doi: 10.15587/2313-8416.2015.38848

45. Doniach, S. Green's Functions for Solid State Physicists [Text] / S. Doniach, E. H. Sondheimer. - College Press London, 1998. - 336 p. doi: 10.1142/p067

46. Blanter, Ya. M. Shot noise in mesoscopic conductors [Text] / Ya. M. Blanter, M. Büttiker // Physics Reports. -2000. - Vol. 336, Issue 1-2. - P. 1-166. doi: 10.1016/s0370-1573(99)00123-4

47. Hasan, M. Z. Colloquium: Topological insulators [Text] / M. Z. Hasan, C. L. Kane // Reviews of Modern Physics. - 2010. - Vol. 82, Issue 4. - P. 3045-3067. doi: 10.1103/ revmodphys.82.3045

48. Кругляк, Ю. А. Расчет электронных оболочек бензильного радикала неограниченным методом Хартри -Фока на гауссовом базисе [Текст] / Ю. А. Кругляк, Х. Прейсс, Р. Яношек // Ж. структ. хим. - 1971. - Т. 12, № 4. - С. 689-696.

49. Кругляк, Ю. А. Обобщенный метод Хартри -Фока и его версии: от атомов и молекул до полимеров [Текст] / Ю. А. Кругляк // ScienceRise. - 2014. - Т. 5, № 3 (5). - C. 6-21. doi: 10.15587/2313-8416.2014.30726

50. Kruglyak, Y. A. Study of the electronic structure of alternant radicals by the DODS method [Text] / Y. A. Kruglyak, I. I. Ukrainsky // International Journal of Quantum Chemistry. - 1970. - Vol. 4, Issue 1. - P. 57-72. doi: 10.1002/qua.560040106

51. Kruglyak, Yu. A. Quantum-chemical studies of quasi-one-dimensional electron systems. 1. Polyenes [Text] /

Yu. A. Kruglyak // ScienceRise. - 2015. - Vol. 5, Issue 2 (10). -P. 69-105. doi: 10.15587/2313-8416.2015.42643

52. Рашба, Э. И. Свойства полупроводников с петлей экстремумов. I. Циклотронный и комбинированный резонанс в магнитном поле, перпендикулярном плоскости петли [Текст] / Э. И. Рашба // ФТТ. - 1960. - Т. 2, № 6. -С. 1224-1238.

53. Bychkov, Yu. A. Oscillatory effects and the magnetic susceptibility of carriers in inversion layers [Text] / Yu. A. Bychkov, E. I. Rashba // Journal of Physics C: Solid State Physics. - 1984. - Vol. 17, Issue 33. - P. 6039-6045. doi: 10.1088/0022-3719/17/33/015

54. Winkler, R. Spin-Orbit Coupling Effects in Two-Dimensional Electron and Hole Systems [Text] / R. Winkler. -Berlin: Springer, 2003. - 228 p. doi: 10.1007/b13586

55. Hanle, W. Uber magnetische Beeinflussung der Polarisation der Resonanzfluorescenz [Text] / W. Hanle // Zeitschrift for Physik. - 1924. - Vol. 30, Issue 1. - P. 93-105. doi: 10.1007/bf01331827

56. Van Dyck, R. The Hanle Effect [Text] / R. Van Dyck, J. Stoltenberg, D. Pengra. - Washington: The University of Washington, 2006.

57. Huang, B. Geometric dephasing-limited Hanle effect in long distance lateral silicon spin transport devices [Text] / B. Huang, H.-J. Jang, I. Appelbaum // Applied Physics Letters. - 2008. - Vol. 93, Issue 16. - P. 162508. doi: 10.1063/ 1.3006333

58. Koo, H. C. Control of Spin Precession in a Spin-Injected Field Effect Transistor [Text] / H. C. Koo, J. H. Kwon, J. Eom, J. Chang, S. H. Han, M. Johnson // Science. -2009. - Vol. 325, Issue 5947. - P. 1515-1518. doi: 10.1126/ science.1173667

59. Wunderlich, J. Spin Hall Effect Transistor [Text] / J. Wunderlich, B.-G. Park, A. C. Irvine, L. P. Zarbo, E. Roz-kotova, P. Nemec et. al. // Science. - 2010. - Vol. 330, Issue 6012. - P. 1801-1804. doi: 10.1126/science.1195816

60. Sih, V. Generating Spin Currents in Semiconductors with the Spin Hall Effect [Text] / V. Sih, W. H. Lau, R. C. Myers, V. R. Horowitz, A. C. Gossard, D. D. Awschalom // Physical Review Letters. - 2006. - Vol. 97, Issue 9.

doi: 10.1103/physrevlett.97.096605

61. Fundamentals of nanoelectronics - quantum models [Electronic resource]. - Available at: http://nanohub.org/ courses/FoN2

62. PurdueX [Electronic resource]. - Free online courses from Purdue University. - Available at: https://www.edx.org/ school/purduex

References

1. Hall, E. H. (1879). On a New Action of the Magnet on Electric Currents. American Journal of Mathematics, 2 (3), 287-292. doi: 10.2307/2369245

2. Klitzing, K. v., Dorda, G., Pepper, M. (1980). New Method for High-Accuracy Determination of the Fine-Structure Constant Based on Quantized Hall Resistance. Physical Review Letters, 45 (6), 494-497. doi: 10.1103/physrevlett.45.494

3. fon Klitcing, K. (1985). Kvantovyj jeffekt Holla: Nobelevskie lekcii po fizike. UFN, 150 (1), 107-126.

4. Reedtz, G. M., Cage, M. E. (1987). An automated potentiometric system for precision measurement of the quan-

tized hall resistance. Journal of Research of the National Bureau of Standards, 92 (5), 303-310. doi: 10.6028/jres.092.030

5. Tsui, D. C., Stormer, H. L., Gossard, A. C. (1982). Two-Dimensional Magnetotransport in the Extreme Quantum Limit. Physical Review Letters, 48 (22), 1559-1562. doi: 10.1103/physrevlett.48.1559

6. Stepanovskij, Ju. P. (1998). Drobnyj kvantovyj jef-fekt Holla. Jelektromagnitnye javlenija, 1 (3), 427-442.

7. Nagaosa, N., Sinova, J., Onoda, S., MacDonald, A. H., Ong, N. P. (2010). Anomalous Hall effect. Reviews of Modern Physics, 82 (2), 1539-1592. doi: 10.1103/revmodphys.82.1539

8. Dyakonov, M. I., Perel, V. I. (1971). Possibility of orientating electron spins with current. Sov. Phys. JETP Lett., 13, 467.

9. Dyakonov, M. I., Perel, V. I. (1971). Current-induced spin orientation of electrons in semiconductors. Physics Letters A, 35 (6), 459-460. doi: 10.1016/0375-9601(71)90196-4

10. Kane, C. L., Mele, E. J. (2005). Quantum Spin Hall Effect in Graphene. Physical Review Letters, 95 (22). doi: 10.1103/physrevlett.95.226801

11. Srinivasan, S., Sarkar, A., Behin-Aein, B., Datta, S.

(2011). All-Spin Logic Device With Inbuilt Nonrecipro-city. IEEE Transactions on Magnetics, 47 (10), 4026-4032. doi: 10.1109/tmag.2011.2159106

12. Kane, C., Moore, J. (2011). Topological insulators. Physics World, 24 (02), 32-36. doi: 10.1088/2058-7058/24/02/36

13. Datta, S. (2012). Lessons from Nanoelectronics: A New Perspective on Transport. Hackensack, New Jersey: World Scientific Publishing Company, 492. doi: 10.1142/8029

14. Krugljak, Ju. A. (2015). Nanoelectronics «bottom -up»: current generation, generalized Ohm's law, elastic resistors, conductivity modes, thermoelectricity. ScienceRise, 7/2 (12), 76-100. doi: 10.15587/2313-8416.2015.45700

15. Krugljak Ju. O., Krugljak, N. Ju., Striha, M. V.

(2012). Uroky nanoelektroniky: vynyknennja strumu, formu-ljuvannja zakonu Oma i mody providnosti v koncepcii' «zny-zu - vgoru». Sensor. elektr. mikrosyst. tehn., 9 (4), 5-29. Available at: http://dspace.onu.edu.ua:8080/handle/123456789/3643

16. Ashkroft N., Mermin N. (1979). Fizika tverdogo tela. Vol. 1-2. Moscow: Mir, 458; 486.

17. Rashba, E. I. (2003). Spin currents in thermodynamic equilibrium: The challenge of discerning transport currents. Physical Review B, 68 (24). doi: 10.1103/ physrevb.68.241315

18. Buttiker, M. (1988). Symmetry of electrical conduction. IBM Journal of Research and Development, 32 (3), 317334. doi: 10.1147/rd.323.0317

19. Krugljak, Ju. A. (2015). Nanoelectronics «bottom -up»: non-equillibrium Green's functions method, model transport problems and quantum interference. ScienceRise, 9/2 (14), 41-72. doi: 10.15587/2313-8416.2015.48827

20. Krugljak, Ju. A. (2015). Nanoelectronics «bottom -up»: non-equillibrium Green's functions method, model transport problems and quantum interference. ScienceRise, 2/2 (7), 93-106. doi: 10.15587/2313-8416.2015.36443

21. Krugljak, Ju. A. (2015). The «bottom - up» nanoe-lectronics: elements of spintronics and magnetronics. ScienceR-ise, 8/2 (13), 51-68. doi: 10.15587/2313-8416.2015.47792

22. Sears, F. W., Salinger, G. L. (1975). Thermodynamics, Kinetic Theory, and Statistical Thermodynamics. Boston: Addison-Wesley.

23. Landauer, R. (1957). Spatial Variation of Currents and Fields Due to Localized Scatterers in Metallic Conduction. IBM Journal of Research and Development, 1 (3), 223-231. doi: 10.1147/rd.13.0223

24. Landauer, R. (1970). Electrical resistance of disordered one-dimensional lattices. Philosophical Magazine, 21 (172), 863-867. doi: 10.1080/14786437008238472

25. Landauer, R. (1988). Spatial variation of currents and fields due to localized scatterers in metallic conduction. IBM Journal of Research and Development, 32 (3), 306-316. doi: 10.1147/rd.323.0306

26. Landauer, R. (1996). Spatial variation of currents and fields due to localized scatterers in metallic conduction (and comment). Journal of Mathematical Physics, 37 (10), 5259. doi: 10.1063/1.531590

27. Buttiker, M. (1986). Four-Terminal Phase-Coherent Conductance. Physical Review Letters, 57 (14), 1761-1764. doi: 10.1103/physrevlett.57.1761

28. Sharvin, Ju. V. (1965). Ob odnom vozmozhnom metode issledovanija poverhnosti Fermi. ZhJeTF,. 48 (3), 984-985.

29. Sharvin, Yu. V., Bogatina, N. I. (1969). Investigation of Focusing of Electron Beams in a Metal by a Longitudinal Magnetic Field. Sov. Phys. JETP, 29 (3), 419-423.

30. Imry, Y.; Grinstein, G., Mazenko, G. (Eds.) (1986). In Directions in Condensed Matter Physics. Singapore: World Scientific, 101.

31. Imry, Y., Landauer, R. (1999). Conductance viewed as transmission. Reviews of Modern Physics, 71 (2), 306-312. doi: 10.1103/revmodphys.71.s306

32. Lesovik, G. B., Sadovskij, I. A. (2011). Opisanie kvantovogo jelektronnogo transporta s pomoshh'ju mat-ric rassejanija. UFN., 181 (10), 1041-1096. doi: 10.3367/ ufnr.0181.201110b.1041

iНе можете найти то, что вам нужно? Попробуйте сервис подбора литературы.

33. Stone, A. D., Szafer, A. (1988). What is measured when you measure a resistance? - The Landauer formula revisited. IBM Journal of Research and Development, 32 (3), 384413. doi: 10.1147/rd.323.0384

34. Golizadeh-Mojarad, R., Zainuddin, A. N. M., Klimeck, G., Datta, S. (2008). Atomistic non-equilibrium Green's function simulations of Graphene nano-ribbons in the quantum hall regime. Journal of Computational Electronics, 7 (3), 407-410. doi: 10.1007/s10825-008-0190-x

35. Haug, R. J. (1993). Edge-state transport and its experimental consequences in high magnetic fields. Semicond. Semiconductor Science and Technology, 8 (2), 131-153. doi: 10.1088/0268-1242/8/2/001

36. Cage, M. E. (1997). Current distributions in quantum Hall effect devices. Journal of Research of the National Institute of Standards and Technology, 102 (6), 677-691. doi: 10.6028/jres.102.045

37. Martines-Duart, Dzh. M., Martin-Palma, R. Dzh., Agullo-Rueda, F. (2007). Nanotehnologii dlja mikro- i optojel-ektroniki. Moscow: Tehnosfera, 368.

38. Berger, C., Song, Z., Li, X., Wu, X., Brown, N., Naud, C. et. al (2006). Electronic Confinement and Coherence in Patterned Epitaxial Graphene. Science, 312 (5777), 11911196. doi: 10.1126/science.1125925

39. Brey, L., Fertig, H. A. (2006). Edge states and the quantized Hall effect in graphene. Physical Review B, 73 (19). doi: 10.1103/physrevb.73.195408

40. Peres, N. M. R., Castro Neto, A. H., Guinea, F. (2006). Conductance quantization in mesoscopic graphene. Physical Review B, 73 (19). doi: 10.1103/physrevb.73.195411

41. Abanin, D. A., Lee, P. A., Levitov, L. S. (2006). Spin-Filtered Edge States and Quantum Hall Effect in Graphene. Physical Review Letters, 96 (17). doi: 10.1103/ physrevlett. 96.176803

42. Kubo, R. (1957). Statistical-Mechanical Theory of Irreversible Processes. I. General Theory and Simple Applications to Magnetic and Conduction Problems. Journal of the Physical Society of Japan, 12 (6), 570-586. doi: 10.1143/jpsj.12.570

43. Yourgrau, W., van der Merwe, A., Raw, G. (1982). Treatise on Irreversible and Statistical Thermophysics. Dover, New York.

44. Krugljak, Ju. A. (2015). Model' provodimosti Lan-daujera-Datty-Lundstroma v mikro- i nanojelektronike i transportnoe uravnenie Bol'cmana. ScienceRise, 3/2 (8), 108116. doi: 10.15587/2313-8416.2015.38848

45. Doniach, S., Sondheimer, E. H. (1998). Green's Functions for Solid State Physicists. College Press London, 336. doi: 10.1142/p067

46. Blanter, Y. M., Büttiker, M. (2000). Shot noise in mesoscopic conductors. Physics Reports, 336 (1-2), 1-166. doi: 10.1016/s0370-1573(99)00123-4

47. Hasan, M. Z., Kane, C. L. (2010). Colloquium: Topological insulators. Reviews of Modern Physics, 82 (4), 3045-3067. doi: 10.1103/revmodphys.82.3045

48. Krugljak, Ju. A., Prejss, H., Janoshek, R. (1971). Raschet jelektronnyh obolochek benzil'nogo radikala neogranichennym metodom Hartri-Foka na gaussovom bazise. Zh. strukt. him., 12 (4), 689-696.

49. Krugljak, Ju. A. (2014). Generalized Landauer-Datta-Lundstrom model of electron and heat transport for micro- and nanoelectronics. ScienceRise, 5/3 (5), 6-21. doi: 10.15587/2313-8416.2014.30726

50. Kruglyak, Y. A., Ukrainsky, I. I. (1970). Study of the electronic structure of alternant radicals by theDODS method. International Journal of Quantum Chemistry, 4 (1), 57-72. doi: 10.1002/qua.560040106

51. Kruglyak, Yu. A. (2015). Quantum-chemical studies of quasi-one-dimensional electron systems. 1. Polyenes. ScienceRise, 5/2 (10), 69-105. doi: 10.15587/2313-8416.2015.42643

52. Rashba, E. I. (1960). Properties of semiconductors with an extremum loop . 1. Cyclotron and combinational resonance in a magnetic field perpendicular to the plane of the loop. Sov. Phys. Solid. State., 2, 1109.

53. Bychkov, Y. A., Rashba, E. I. (1984). Oscillatory effects and the magnetic susceptibility of carriers in inversion layers. Journal of Physics C: Solid State Physics, 17 (33), 6039-6045. doi: 10.1088/0022-3719/17/33/015

54. Winkler, R. (2003). Spin-Orbit Coupling Effects in Two-Dimensional Electron and Hole Systems. Berlin: Springer, 228. doi: 10.1007/b13586

55. Hanle, W. (1924). Uber magnetische Beeinflussung der Polarisation der Resonanzfluoreszenz. Zeitschrift for Physik, 30 (1), 93-105. doi: 10.1007/bf01331827

56. Van Dyck, R., Stoltenberg, J. Pengra, D. (2006). The Hanle Effect. - Washington: The University of Washington.

57. Huang, B., Jang, H.-J., Appelbaum, I. (2008). Geometric dephasing-limited Hanle effect in long-distance lateral silicon spin transport devices. Applied Physics Letters, 93 (16), 162508. doi: 10.1063/1.3006333

58. Koo, H. C., Kwon, J. H., Eom, J., Chang, J., Han, S. H., Johnson, M. (2009). Control of Spin Precession in a Spin-Injected Field Effect Transistor. Science, 325 (5947), 15151518. doi: 10.1126/science.1173667

59. Wunderlich, J., Park, B.-G., Irvine, A. C., Zarbo, L. P., Rozkotova, E., Nemec, P et. al. (2010). Spin Hall Effect Transistor. Science, 330 (6012), 1801-1804. doi: 10.1126/ science.1195816

60. Sih, V., Lau, W. H., Myers, R. C., Horowitz, V. R., Gossard, A. C., Awschalom, D. D. (2006). Generating Spin Currents in Semiconductors with the Spin Hall Effect. Physical Review Letters, 97 (9). doi: 10.1103/physrevlett.97.096605

61. Fundamentals of nanoelectronics - quantum models. Available at: http://nanohub.org/courses/FoN2

62. PurdueX. Free online courses from Purdue University. Available at: https://www.edx.org/school/purduex

Рекомендовано до публгкацИ д-р ф1з.-мат. наук Глушков О. В.

Дата надходження рукопису 21.09.2015

Кругляк Юрий Алексеевич, доктор химических наук, профессор, кафедра информационных технологий, Одесский государственный экологический университет, ул. Львовская, 15, г. Одесса, Украина, 65016

E-mail: quantumnet@yandex.ua

i Надоели баннеры? Вы всегда можете отключить рекламу.